at - 5 - zohowinlearning.zohosites.com/files/pdf/applied technology...training materials and has...

140
Applied Technology Level 5 Worldwide Interactive Network, Inc. 1000 Waterford Place, Kingston, TN 37763 • 888.717.9461 ©2008 Worldwide Interactive Network, Inc. All rights reserved.

Upload: others

Post on 28-Aug-2020

1 views

Category:

Documents


0 download

TRANSCRIPT

Applied TechnologyLevel 5

Worldwide Interactive Network, Inc.1000 Waterford Place, Kingston, TN 37763 • 888.717.9461

©2008 Worldwide Interactive Network, Inc. All rights reserved.

2 • Applied Technology

Copyright © 1998 by Worldwide Interactive Network, Inc. ALL RIGHTS RESERVED.Printed in the U.S.A. No part of this publication may be reproduced, stored in a retrievalsystem, or transmitted in any form or by any means, electronic, photocopying, recordingor otherwise without the prior written permission of Worldwide Interactive Network,Inc.

ACT™ and WorkKeys® are trademarks of ACT, Inc. Worldwide Interactive Network,Inc. is not owned or controlled by ACT, Inc.; however, ACT, Inc. has reviewed thesetraining materials and has determined that they meet ACT, Inc.’s standards for WorkKeysTraining curriculum. The WorkKeys employment system is a product of ACT, Inc.

The use of materials in this manual does not imply any specific results when WINmaterials are used with the ACT WorkKeys system.

Requests for permission to reproduce or make other use of this material should beaddressed to:

Worldwide Interactive Network, Inc.1000 Waterford PlaceKingston, Tennessee 37763 USATel: (865) 717-3333Fax: (865) [email protected]

Applied Technology • 3

Hi! Let me introduce myself to you. I am EdWIN,and I will be your friendly guide through this study.Now, you may have met me before in a different courseor in an earlier level. If you have, I hope that you havefound that I am not too tough of a taskmaster. Lookfor me to pop up every now and then with a tip, hint,or maybe even a quiz question or two!

You are about to begin Level 5 of Applied Technology.If you have already completed Level 4, you know thatit wasn’t too tough. Even if you are a “technophobe,”you probably found that it wasn’t as difficult as youthought it might be.

We will be studying thermodynamics, fluiddynamics, electricity, and mechanics in this course. Thislevel will cover the basics once again for your review.The main difference will be that the exercises will bemore difficult. The great thing about this type oflearning is that you generally can set your own pace.Consequently, you will not have to move on until youfeel you have learned the material to your satisfaction.

So, if you are ready, so am I. Turn the page and let’sbegin!

INTRODUCTION

Hi, I’m Edwin.

4 • Applied Technology

Problem-Solving Strategies

The basic components of effective problem-solving strategies are –

• Identifying the problem (“What is the goal?” and “What limits does the goalimpose?”)

• Analyzing and interpreting data (reading a gauge, interpreting a printout)This includes identifying and disregarding nonessential data.

• Exploring and evaluating solutions (“What options are available?” and “Whichoption is best, taking into account many variables, including cost, time, humanresources, materials, environment, and expertise?”)

One well-known problem-solving model is the IDEAL* model. The IDEAL modelwas designed as an aid for teaching and improving problem-solving skills. TheIDEAL process includes the following steps:

I = Identify the problem (determine what needs to be done).D = Define and represent the problem (sharpen and clarify the boundaries).E = Explore alternative approaches (analyze and evaluate alternatives).A = Act on a plan (determine the logical steps to be used and how to progress

through the steps).L = Look at the result (determine whether or not the plan worked).

The exercises in this workbook guide learners through the IDEAL problem-solvingprocess.

* The IDEAL Problem Solver: A Guide for Improving Thinking, Learning, and Creativity , © 1984. Permission for use granted by W.H. Freeman and Company/Worth Publishers, all rights reserved.

STRATEGIES

Applied Technology • 5

Problem-solving strategies are critical to any instruction aimed at improving alllevels of applied technology skills. To be an effective technological problem solver,you need to be able to do the following:

• Understand cause-effect relationships (What parts of systems affect and areaffected by other parts?)

• Make comparisons (What commonalities and differences do systems have?)• Recognize probable outcomes (How will the system react to a specific action?)• Predict what should happen next (Based on what has been observed, what is

known about a specific system, and what is known about related scientificprinciples, make a prediction about what will happen next.)

• Judge spatial relationships (Visualize how a system operates and mentally rotatesystem parts to solve problems within a given system.)

• Notice what appears out of place (Observe a malfunctioning system in operationto determine what is not working correctly.)

Course Strategies

The best way for me to help you be an effective problem solver is to give youopportunities to develop and refine your problem-solving skills. Therefore, I willuse the following strategies:

• I will minimize instructions so that you are encouraged to invent innovativeways to accomplish the tasks.

• I will provide you with a variety of materials from which to choose.• If you have questions about the exercises, reread the beginning instructions.

This strategy encourages you to figure out how to use the materials to reachyour goal.

• You have plenty of time to explore. As long as you are actively engaged, learningis taking place. If ample time is allowed, you will be able to do more in-depthinvestigation. Thinking about a problem is part of learning.

• I encourage you to share ideas with others. This strategy reflects how peoplesolve problems in the workplace — with input from others. Most problem-solving activities lend themselves to having you work individually, but you areencouraged to seek others’ input.

STRATEGIES

6 • Applied Technology

BASIC SCIENTIFIC PRINCIPLES

Applied Technology focuses on:

• Principles related to power sources – for thermal,fluid, electrical, and mechanical systems.

• Principles related to flow – for thermal, fluid,electrical, and mechanical systems.

• Principles related to pressure – for thermal, fluid,electrical, and mechanical systems.

• Principles related to resistance – for thermal, fluid,electrical, and mechanical systems.

Some basic scientific principles involved with energysources, flow, pressure, and resistance follow:

Bernoulli’s principle: The faster the flow of air or fluid, the lower the pressure.

Boyle’s law: For a certain amount of gas, at a constant temperature, as the pressure (P)increases, the volume (V) of the gas decreases so that P times V is constant (k). (PV=k).

Charles’ law: For a certain amount of gas, at a constant pressure, as the absolutetemperature of the gas increases, the volume of the gas also increases. Mathematicallythis is: Volume (V) divided by temperature (T) equals a constant (k). V/T=k Thetemperature must be on an absolute scale that is in reference to absolute zero.

Hooke’s law: The greater the force exerted on an object, the more it will be moved. Forexample, the heavier the weight hanging from a spring, the more the spring will bestretched.

Newton’s laws of motion:• An object will remain at rest or in uniform motion unless acted upon by an

outside force.• When a force acts upon an object, it changes the momentum of that object,

and this change is proportional to the applied force and to the time that it actsupon the object.

• Every action (force) is followed by an equal and opposite reaction (force).

Ohm’s law: Current is directly proportional to the voltage and inversely proportional tothe resistance.

Pascal’s law: Pressure added to a confined fluid at any point instantly appears equally atall other points, and is always at right angles to the confining surfaces.

BASIC PRINCIPLES

Applied Technology • 7

LESSON 1 Thermodynamics

LESSON 2 Fluid Dynamics

LESSON 3 Electricity

LESSON 4 Mechanics

LESSON 5 Posttest

REFERENCES Test-Taking TipsBasic Scientific PrinciplesBibliographyAnswers to Pop Quiz Questions

OUTLINE

8 • Applied Technology

THERMODYNAMICS

Thermodynamics is a very complicated science.There is a lot that can be learned about it. We are goingto review the basic concept of the topic and practicebasic problems using our acquired knowledge. Theproblems at this level will be more difficult. But, don’tworry; the principles are the same.

Remember, all matter is made up of particles thatare in constant motion. This motion manifests itself asthe form of energy called heat. The study of heat iscalled thermodynamics. Now, that wasn’t so hard, wasit? There are two basic laws of thermodynamics.

Basic Laws of Thermodynamics

1. Energy cannot be created or destroyed.

2. Heat energy always flows spontaneously fromhot to cold.

LESSON 1

My mind spontaneouslyflows from cold to hot!

Applied Technology • 9

Some other generalizations that can be made about heatinclude:

• Heat travels through conductors such as metal betterthan it travels through insulators such as brick orwood. For example, you know that if you stir boilingwater with a metal spoon, it will soon heat up tothe point where you cannot touch it. If you stirboiling water with a wooden spoon, you can stir itindefinitely without it getting too hot to touch.

• Dark-colored surfaces absorb more heat than light-colored surfaces.

• Rough or dull surfaces absorb more heat thansmooth or shiny surfaces.

• When friction causes heat, the object that is inconstant contact gets hotter than the movableobject.

In this level we will be concentrating on heatingand cooling systems. Several scientific principles areinvolved in these problems including airflow, resistance,equilibrium, and fluid pressure.

Heating and cooling systems involve a variety ofdevices including compressors, shutoff valves,thermostats, filters, ductwork, and diffusers. Thediffusers, which are usually in the ceiling or floor, directair into the room.

LESSON 1

10 • Applied Technology

You may think that you are totally unfamiliar withthese systems, but you probably know more than youthink! Many homes have central heat and air units.These units have the previously mentioned parts. Youprobably work with these systems often. For example,every time you change the setting on your thermostat,you are manipulating your system. If you open or closea vent, you are making a change in the diffuser. If yoursystem quits working, you hope and pray that themaintenance person you call doesn’t tell you that yourcompressor must be replaced! Silver-colored ductworkthat is in your attic or basement directs the flow of airinto each room. What happens when your thermostatis not functioning properly? Look out when you receiveyour next electric bill! See, you do know more about itthan you thought.

Let’s look at the diagram of a cooling system thatuses a radiator instead of a compressor to provide thecooling function.

As air passes over the radiator, it will be cooled bythe water circulating in and out of the radiator. Theblower motor forces the air through the ductwork. Airvalves, or dampers, are adjustable to regulate the airflowinto each room. In this diagram, there is one thermostatwhich is set at 72 degrees. All the rooms should be atthat temperature. The volume, or total amount of air,determines the actual room temperature.

LESSON 1

Applied Technology • 11

Diagram A

For example, if you have one of the vents wide-open in one room and one half-open in another, thetemperature in those rooms will be different. Also, alow water level in the radiator would affect all fourrooms. Equilibrium is achieved when the airflow andresulting temperatures even out in all four rooms towithin one degree.

Think about this process as you complete thefollowing exercise.

LESSON 1

12 • Applied Technology

IDEAL

EXERCISE – OFFICE COOLING AND HEATING

Instructions: Read the following scenario and refer to Diagram A. Then, answer the questionspertaining to it.

Scenario

In the springtime, four offices that are controlled by the sameair-conditioning system are at different temperatures (asindicated on the previous diagram). Some rooms havewindows, and some do not. One thermostat controls all fourrooms. As the diagram shows, each room has ductwork goingto it, which ends in an air diffuser in the ceiling of that room.There is a circuit breaker to prevent electrical overload onthe system. Your job is to equalize the temperature in thefour rooms.

Identify the problem

1. What is the problem?

a. The water going into the radiator is too hot.b. The temperature is different in each of the four rooms.c. There are not enough windows.d. The air ducts are the wrong size.e. There are too many cold air ducts.

LESSON 1

Applied Technology • 13

2. The problem with the system could be that:

a. the water level is too low in the system.b. the circuit breaker has tripped.c. the blower system is not working correctly.d. the sun coming in the window is creating the malfunction.e. the thermostat is not operating.

Define the problem

3. To identify the malfunction, first check:

a. the water pressure.b. the thermostat setting.c. the bearing on the blower motor.d. to see whether air is coming out of the diffusers in each room.e. the circuit breaker.

Explore alternatives

4. To isolate the problem, you would first:

a. measure airflow coming from the diffuser in each room.b. measure the pressure drop across each filter.c. check the water temperature before it enters and after it exits the

radiator.d. check the voltage going to the thermostat.e. measure the temperature outside the building.

LESSON 1

14 • Applied Technology

Act on a plan

5. Which of the following measurements taken in the system wouldindicate an existing problem?

a. The pressure drop across Filter A is 2 psi (pounds per squareinch); the pressure drop across Filter B is 2 psi.

b. Air leaving the radiator is 68°F; the temperature of the return-airduct is 76°F.

c. The airflow rate to each room is:Room 1 – 60 cfm (cubic feet per minute)Room 2 – 100 cfmRoom 3 – 80 cfmRoom 4 – 85 cfm

d. The motor speed is 1,700 rpm.e. The outside relative humidity is 90%.

Look at the result

6. To equalize the airflow, you should change the airflow by:

a. opening D1 and closing D2 slightly.b. opening D3 and D4.c. closing D3 and D4.d. closing D1 and opening D2 slightly.e. opening D2, D3, and D4.

LESSON 1

Applied Technology • 15

ANSWERS TO EXERCISE

1. What is the problem?

Answer: b. The temperature is different in each of the four rooms.

2. The problem with the system could be that:

Answer: c. the blower system is not working correctly.

3. To identify the malfunction, first check:

Answer: d. to see whether air is coming out of the diffusers in eachroom.

4. To isolate the problem, you would first:

Answer: a. measure airflow coming from the diffuser in each room.

IDEAL

LESSON 1

16 • Applied Technology

5. Which of the following measurements taken in the system wouldindicate an existing problem?

Answer: c. The airflow rate to each room is:Room 1 – 60 cfmRoom 2 – 100 cfmRoom 3 – 80 cfmRoom 4 – 85 cfm

6. To equalize the airflow, you should change the airflow by:

Answer: a. opening D1 and closing D2 slightly.

How did you do on that problem? Did youunderstand the answers? If not, go back and study thediagram and reread the answers to figure out theprocess. When you are satisfied that you understandit, go on to the next exercise.

LESSON 1

Applied Technology • 17

This next exercise will deal with factors that affectthe flow of air, such as heat, resistance, and the effectsof fuses and breakers.

We discussed the purpose of fuses and breakers inan earlier level of Applied Technology while discussingelectricity. You will remember that fuses and breakersare safety devices that prevent an overload on anelectrical circuit. Overload can cause “short outs” whichcan result in fire. Subsequently, you can see how fusesand breakers would be an important device in anysystem!

Fuses and circuit breakers also can serve otherpurposes. In the next example, the purpose of the fusesand breakers is to stop the electrical current when thepreset temperature is exceeded, therefore turning offthe device and preventing overheating.

You will often encounter breakdowns in equipmentwhen poor ventilation exists in and around equipment.Poor ventilation will cause the equipment to overheat,and therefore cause damage to it.

Have you ever noticed the vents in your VCR,television, radio, or other common appliance? If youread the guidelines that came with the device, youshould have noticed a warning specifically stating NOTto cover the vents and to allow space around the device.

LESSON 1

Pop Quiz:Name the steps of theIDEAL problem-solvingstrategy.

18 • Applied Technology

<

This is because the manufacturer wants to preventoverheating of the equipment. If you have made themistake of laying the newspaper or videotapes on topof the machine, you know what can happen! … Beforelong, the machine may overheat, causing a malfunction.Many times the machine will be ruined completely.Heat will accumulate, even in ventilated areas, if theequipment achieves the maximum level of activity.

Therefore, it is important for you to consider theseconditions, particularly in enclosed spaces. You mustbe able to place equipment or system componentswithin the constraints of the workplace environmentand prevent problems with overheating. You may findyourself in a work environment where this knowledgewill be vital.

Before we proceed to the next exercise, let’s considersome learning activities that will help you understandsome of these principles and encourage critical thinkingon the properties of airflow.

Thinking Activity

Suppose you have two thermometers and a fan. Place one of thethermometers directly into the flow of air and one below the fan where itdoes not catch the breeze. What do you think will happen to the temperatureof each thermometer?

Answer: The temperatures will not change.

Did that answer surprise you? Perhaps you thoughtthat the breeze would cool the thermometer that wasplaced in front of it. Why not?

LESSON 1

Applied Technology • 19

<

<

That is because a fan simply moves the existing air.It does not change the temperature of the air. This is abit of an illusion since everyone knows that standingin front of a fan on a hot day seems to cool you. Actually,just the movement of the air around your body feelscool to you because your body temperature is generallyhigher than the air that is being moved. Part of this isdue to moisture that is on your body and other physicalfactors.

Thinking Activity

Suppose you hold a smoking match below a fan so the smoke rises into thepath of the breeze. How do you think the smoke will move?

Answer:The smoke will move away from the fan.

Once again, this is because the air is being movedoutwardly by the movement of the blades. The smokewill naturally go in the direction that the air is flowing.

Thinking Activity

Now suppose you hold the smoking match directly below or behind thefan. In which direction do you think the smoke will be moved?

Answer: The smoke will be drawn up into the fan.

That is because the air behind and below the fan ispulled up into the fan and then blown outward. Now,let’s think about two thermometers.

LESSON 1

20 • Applied Technology

<

< Thinking Activity

Suppose you wrap one of the thermometers in a wet cloth. If you hold boththermometers in front of the fan, what do you think will happen to thetemperature of the thermometers?

Answer: The thermometer that has the wet cloth around it will be cooled.

The other thermometer will not change. Why doesthis happen? The airflow created by the fan carries awaywater vapor from the wet cloth. The creation of thiswater vapor consumes heat, which is pulled throughthe cloth from the thermometer. The heat removedfrom the thermometer is shown as a reduction in itstemperature. Let’s consider an activity using heat toillustrate airflow.

Thinking Activity

Suppose you have a beaker of water placed over a Bunsen burner (heatsource). What will happen?

Answer: The heat source will cause the heated water to boil causing the vaporto rise to the top of the container.

The heat source will increase the temperature ofthe water. At some temperature, the vapor pressure ofthe water will be greater than the atmospheric pressure.At this point, the water liquid turns into water vaporbubbles. This is “boiling.” The water vapor bubblesrise in the water since they are less dense than thesurrounding liquid water.

OK, now that we have done some critical thinkingconcerning airflow, let’s solve some problems.

LESSON 1

Applied Technology • 21

EXERCISE – TRANSFORMER BOX FANS

Instructions: Read the following scenario and study the diagram. Answer the questionspertaining to it.

Scenario

In a factory where you work, the plant temperature ismaintained at 25 degrees Celsius. An electrical box that ismounted on the wall in the plant contains a bank of breakersand fuses in its top portion. Two transformers are located inthe bottom area of the box. The breakers and fuses aretemperature sensitive, rated up to 40 degrees Celsius. Eachtransformer can reach a temperature of 140 degrees Celsiuswithout being damaged. The box’s location, which ispermanent, requires the fans to be inside the box. Your job isto install fans inside the box to maintain a safe operatingtemperature for the components.

LESSON 1

22 • Applied Technology

Identify the problem

1. What is your assignment?

a. to determine the correct position of the fans to keep componentsfrom overheating

b. to determine where the electrical box should be placed in theplant

c. to determine what 140 degrees Celsius is in Fahrenheitd. to determine whether the box size is correct for the plante. to determine whether the vents are arranged correctly

Define the problem

2. What is the MOST important factor to consider in order to solve thisproblem?

a. the size of the breakers and fusesb. the size of the fansc. the optimal airflow from the fansd. the size of the ventse. whether the metal of the electrical box is expanding or contracting

from the heat

Explore alternatives

3. What is the BEST alternative for keeping the components fromoverheating?

a. Arrange the angles of the vents.b. Open the doors in the plant.c. Turn one transformer off.d. Arrange the fans to circulate air from the outside of the electrical

box through the box.e. Increase the fuse size.

LESSON 1

Applied Technology • 23

Act on a plan

4. How should the fans be arranged for optimum airflow?

a. The fans should be eliminated because they are producing heat.b. The fans should be facing each other and should be placed at

either end of the electrical box.c. The fans should be positioned so they point directly out of each

vent on one side of the box.d. Both fans should draw the warm air upward over the breakers

and fuses to exit the top of the electrical box.e. Both fans should draw the cool air downward over the breakers

and fuses to push the warmer air out of the bottom of the electricalbox.

Look at the result

5. Once you install the fans, are the components overheating?

a. yesb. no

IDEAL

LESSON 1

24 • Applied Technology

ANSWERS TO EXERCISE

1. What is your assignment?

Answer: a. to determine the correct position of the fans to keepcomponents from overheating

2. What is the MOST important factor to consider in order to solve thisproblem?

Answer: c. the optimal airflow from the fans

3. What is the BEST alternative for keeping the components fromoverheating?

Answer: d. Arrange the fans to circulate air from the outside of theelectrical box through the box.

4. How should the fans be arranged for optimum airflow?

Answer: c. The fans should be positioned so they point directly out ofeach vent on one side of the box.

LESSON 1

Applied Technology • 25

5. Once you install the fans, are the components overheating?

Answer: b. no

How did you do on that exercise? If you missedsome of the questions, go back and review before yougo on to the next exercise.

LESSON 1

26 • Applied Technology

The next exercise will involve factors that affectairflow, including heat and resistance. Let me give youa little background prior to this exercise.

Because the manufacturing process often leaves by-products that flow into the air, many manufacturingplants have air filtration systems. If you are employedin this type of environment, you may need to knowhow to identify malfunctions in air filtration systemsand how to fix them.

LESSON 1

CAUTION

Beware of malfunctioning air filtration systems.

Applied Technology • 27

EXERCISE – SMOG HOG

Instructions: Read the scenario and refer to the following diagram to answer the questionsabout this problem.

Scenario

Your plant produces plastics. Exhaust fumes from theproduction process are collected by fume hoods and aretreated in a device called a Smog Hog. The Smog Hog mustbe kept at a high temperature to operate properly. The heat inthe system is input by a forced-air furnace.

It is your job to oversee the operation of the exhaust treatmentprocess. You notice that the temperature of the Smog Hog(Temp. B) has dropped below the required operatingtemperature. You must correct the problem.

LESSON 1

28 • Applied Technology

Identify the problem

1. What have you been asked to do?

a. recondition the airb. lower the temperature in the roomc. raise the temperature of the Smog Hogd. lower the temperature of the Smog Hoge. call your supervisor

Define the problem

2. The problem is that:

a. the exhaust blower is not functioning.b. the operating temperature is too low.c. no fumes are being collected.d. fume hood 2 is not functioning.e. the operating temperature is too high.

3. To locate the source of the problem, which of the following shouldyou check FIRST?

a. the blower motorb. the natural gas supply to the furnacec. the temperature at Temp. A, 1, 2, 3, and 4d. the furnace filtere. the exhaust air

LESSON 1

Applied Technology • 29

Explore alternatives

4. What would be the indication of the source of the problem?

a. The furnace filter is clogged.b. The blower motor is burned up.c. The temperatures are as follows: Temp. 1 is 130˚F; Temp. 2 is 135˚F;

Temp. 3 is 81˚F; Temp. 4 is 129˚F.d. The exhaust blower motor’s speed is 1,750 rpm.e. The inlet air is hot.

Act on a plan

5. What would you do?

a. Open D1 slightly.b. Remove the furnace filter.c. Turn up the furnace temperature.d. Close D3 slightly.e. Open D2 slightly.

Look at the result

6. After taking action, what would you expect to happen?

a. Temp. A and Temp. 2 will increase.b. Temp. B and Temp. 3 will increase.c. The blower motor’s load will go down.d. The airflow out of the exhaust will increase.e. The furnace will turn off.

LESSON 1

30 • Applied Technology

ANSWERS TO EXERCISE

1. What have you been asked to do?

Answer: c. raise the temperature of the Smog Hog

2. The problem is that:

Answer: b. the operating temperature is too low.

3. To locate the source of the problem, which of the following shouldyou check FIRST?

Answer: c. the temperature at Temp. A, 1, 2, 3, and 4

LESSON 1

Applied Technology • 31

LESSON 1

4. What would be the indication of the source of the problem?

Answer: c. The temperatures are as follows: Temp. 1 is 130˚F; Temp. 2is 135˚F; Temp. 3 is 81˚F; Temp. 4 is 129˚F.

5. What would you do?

Answer: d. Close D3 slightly.

6. After taking action, what would you expect to happen?

Answer: b. Temp. B and Temp. 3 will increase. Excessive outside air isbeing drawn through P3 – This has a cooling effect onboth Temp. 3 and ultimately on Temp. B.

32 • Applied Technology

#

FLUID DYNAMICS

Before we begin this lesson, let’s review some of thebasics that we discussed in earlier levels. This shouldreinforce principles if you have been working with mein previous levels of Applied Technology. If you arestarting at this level, this will help you to rememberthe basics.

Some generalizations that can be made about fluidsare provided:

Concerning pressure

• The amount of pressure exerted by a fluiddepends upon the height and the density ofthat fluid and is independent of the shape ofthe container that is holding the fluid.

• The deeper the fluid, the greater the pressureit exerts.

• The denser the fluid, the greater the pressureit exerts (e.g., salt water is denser than freshwater).

• Fluids seek equilibrium – they seek their ownlevel; a fluid will flow from a place of highpressure to a place of low pressure.

• A fluid can never rise higher than its sourcewithout an external force (e.g., a pump).

LESSON 2

Ready to study fluiddynamics?

Applied Technology • 33

_Q

Concerning evaporation

• The higher a liquid’s temperature, the fasterit will evaporate.

• The lower a liquid’s pressure, the faster theliquid will evaporate.

• The more area of a liquid that is exposed toair, the faster the liquid will evaporate.

• The more circulation of air above a liquid,the faster the liquid will evaporate.

Concerning boiling point

• Increased pressure on a liquid raises theliquid’s boiling point.

• Decreased pressure on a liquid lowers theliquid’s boiling point.

Principles of fluid dynamics include flow, pneumatic(air) pressure, and hydraulic (water or oil) pressure.Pressure can be applied in many different ways to dowork. For example, a water wheel can turn a grinder togrind corn into meal. This principle was appliedthousands of years ago to help people accomplish workmuch easier and faster. Before the invention of the waterwheel, people had to grind corn by hand to make meal.Hand grinding also applied to wheat in order to makeflour. Just think what a work-saving device the waterwheel became to our ancestors! It allowed them to domore in less time and with much less effort.

LESSON 2

34 • Applied Technology

Most of us have seen an old fashioned water wheel.At a glance, you may not think that there is any pressureinvolved in the operation. The water usually flowsthrough a flume until it reaches the wheel, where itdrops onto it, causing it to turn. The wheel then isattached to large gears which also turn to provide thegrinding effect. You can reach down and put your handinto the water just as it is falling onto the wheel andfeel the pressure of the water forcing your hand down.Simple gravity is the basis of this machine.

This basic principle can also be applied usingcompressed air, oil, or water to provide extremepressure. This type of pressure could be dangerous ifyou were to be in a direct “line of fire” so to speak.

A good example of this type of compressed pressureis a nail gun. You may have seen the force with whichthe nail is driven into a board. The power is providedby compressed air or fluid. I don’t have to explain thedanger of being exposed to a machine using this kindof pressure. You certainly would not want to put yourhand in front of the gun! Both of the examples I havegiven use the scientific principle of fluid dynamics tomake them work.

Another type of machine that is powered bycompressed air is an air driver. It is a little morecomplicated than the nail gun, as it needs lubricationin order to operate properly and be durable. To assurethat this happens, each drop of lubrication in the airdriver is fed by a lubricator that is filled with a standardgrade pneumatic oil. There may be more than onedriver in an air gun. Only one lubricator is needed forall the drivers if the drivers do not run at the sametime. Each lubricator is fully adjustable so that driversare provided with proper lubrication.

LESSON 2

Applied Technology • 35

<

<

Let’s consider a couple of learning activities thatshould help you understand the principle of fluiddynamics.

Thinking Activity

Most every child has made a pinwheel at some time in his or her life. Thisis an easy example of air pressure. All it takes is a straw, a straight pin, anda piece of paper cut and folded into a pinwheel. If you release the air froma balloon onto the pinwheel, the pinwheel turns.

The reason for using a balloon to provide the airpressure is to demonstrate that air (energy) can be storedto do work at a later time.

Thinking Activity

Suppose that you have several pieces of plastic PVC pipe cut into differentlengths. By constructing a simple flow pattern with the pipes, you candirect air or water through them in many ways. By using Ts and shutoffvalves, you can direct the air or water into whichever pipe you want. Imaginethat you have a piece of tissue paper attached to one of the ends of the pipe.By blowing through the opening or using a standard bicycle pump, youcan make the tissue flutter at the end. If you want to direct the flow of air,you can shut off one or more of the pipes, and the air will flow along thepipes that have remained open.

Your water pipes at home are constructed this way.Whenever you turn on a faucet, you have opened avalve which allows the water to flow. The water is underpressure so that it can flow uphill.

LESSON 2

36 • Applied Technology

< Thinking Activity

Here’s another easy demonstration of air pressure. Suppose you have anempty dishwashing detergent bottle. Place a small amount of clay over thenozzle end molding it tightly over the opening. Holding the bottle next toyour ear, squeeze gently to check for any air leaks around the top. Then,laying the bottle on the floor, being careful to point it away from anyone,jump on the bottle with all your weight. The piece of clay will be shot outat a great speed and pressure. See why I told you to point it away fromanyone? The clay plug was driven by compressed air.

The same principle can be demonstrated using wateror oil, but it would be much messier! When air, gas, orfluid is compressed, it exerts greater force.

Look at the following examples of compressed air,water, and oil. You will recognize most or all of themas common items used by people all of the time.

LESSON 2

Pop Quiz:As fluid depth increases, does the pressureincrease or decrease?

Applied Technology • 37

Examples that show energy movement:

Air• Aqua-Lung (compressed air and gas)• Vacuum cleaner (reversed airflow)• Windmill (wind-driven water pump)• Forced-air furnace heating system• Pinwheel• Helicopter blades• Air line at gas station (compressed air)

Water• Toilet tank• Fire hose (pressurized water)• Mill wheel (flow of water moving a grinding wheel)• Water pipe in a home (pressurized to allow water to run uphill)• Water tower (holds a large quantity of water as stored energy; allows

water to flow as needed)

Oil• Hydraulic jack• Car shock absorbers• Hatchback closing on a car• Dump truck• Garbage truck• Log splitter

Now that we have gone over some of the principlesconcerning pneumatic and fluid pressure, let’s try aproblem using what we have learned.

LESSON 2

38 • Applied Technology

LESSON 2

EXERCISE – MINI-OILER AIR LINE LUBRICATOR

Instructions: Read the following background information and scenario regarding anautomated teller machine (ATM) production line. Then, refer to the followingdiagram to answer the questions about this problem.

Applied Technology • 39

Mini-Oiler Layout

The diagram shows a filter/regulator/lubricator at every tool station of anATM production line. The line is powered by compressed air. Various toolsneeded in the manufacturing process are attached to the coupler nipples ofthe tool stations.

In order for air-driven tools to last longer and operate accurately, they mustbe lubricated properly. To assure that this happens, the air driver at eachtool station is fed by a lubricator that is filled with oil. Only one lubricatoris needed for three air-driven tools, if the tools are not used at the sametime. Each lubricator is fully adjustable so that drivers are provided withproper lubrication.

Mini-Oiler Filling Instructions

The following steps must be followed to fill the oilers and to keep adequateoil supplied to the air drivers. NOTE: Operators should never let Mini-Oilers get below the empty mark.

• The air lines going into the oiler must be disconnected to performthis activity.

• Inspect the oil base for water, foam, bubbles, etc. If any of thesesubstances are present, contact the maintenance department forrepair.

• Unscrew the filler screw on the top of the Mini-Oiler, under thered cap. Add oil to the full mark. Place the filler screw back onand tighten it.

LESSON 2

40 • Applied Technology

Scenario

You are a trained associate at a plant that produces automatedteller machines. You have been certified in the use of airdrivers, which provide power to the manufacturing line’s air-driven tools. You are responsible for using the appropriatetorque analyzer to check the torque of your air driver on ascheduled basis or when torque valves are suspect. You musttake the average of five readings. You are responsible forrecording the results of your inspection. The torquespecification should be from 14 to 18 in lb (inch pounds).This month’s readings on your air driver are as follows: 13.45in lb, 12.30 in lb, 13.60 in lb, 14.25 in lb, and 12.65 in lb. Inaddition you have noticed excessive heat buildup on the airdriver during the past month. It is your responsibility todetermine what the problem is and to fix it.

Identify the problem

1. Your assignment is to:

a. replace your air driver.b. determine the reason for the heat and low torque readings on the

air driver.c. adjust the torque on your air driver.d. move to another station.e. quit for the day.

LESSON 2

Applied Technology • 41

Define the problem

2. What do you need to consider when troubleshooting this problem?

a. how many workstations are on the same air-supply lineb. whether the torque readings are in the appropriate rangec. the temperature outsided. the grade of oil that is being usede. the humidity outside

Explore alternatives

3. To determine possible explanations for the malfunction, you should:

a. check for pipe leaks.b. check the compressor.c. check the oil lubricator.d. ignore it and assume that the torque analyzer is bad.e. obtain a fan to cool the workstation.

4. Of the following, which would be the most likely cause of theproblem?

a. The relative humidity for the day is extremely high.b. The lubricator lacks oil.c. The are too many workstations off of the main air supply.d. There is a blockage in the system piping.e. You are applying too much pressure to the air driver.

LESSON 2

42 • Applied Technology

LESSON 2

This page is intentionally left blank.

Applied Technology • 43

LESSON 2

Act on a plan

5. What action will you take now?

a. Add a fan to cool the air driver.b. Shut down your station and take the driver to another workstation.c. Add oil to the lubricator.d. Adjust the psi on the compressor.e. Use a hand driver to allow your driver to cool down.

Look at the result

6. After half an hour of using the tool, you:

a. do nothing more and assume that the torque rating is closeenough.

b. do nothing since the tool feels cooler.c. recheck using the torque analyzer to assure that the reading falls

between 14 and 18 in lb.d. visually inspect the hardware to make sure that it is working

properly.e. decide that you need another fan at your workstation.

IDEAL

44 • Applied Technology

ANSWERS TO EXERCISE

1. Your assignment is to:

Answer: b. determine the reason for the heat and low torque readingson the air driver.

2. What do you need to consider when troubleshooting this problem?

Answer: b. whether the torque readings are in the appropriate range

3. To determine possible explanations for the malfunction, you should:

Answer: c. check the oil lubricator.

4. Of the following, which would be the most likely cause of theproblem?

Answer: b. The lubricator lacks oil.

5. What action will you take now?

Answer: c. Add oil to the lubricator.

6. After half an hour of using the tool, you:

Answer: c. recheck using the torque analyzer to assure that thereading falls between 14 and in 18 in lb.

LESSON 2

Applied Technology • 45

EXPLANATION OF ANSWERS

For this exercise, I will provide some explanation about why some choices for the answers areincorrect. I hope this is helpful to you.

1. Correct answer: b

Why not?a. You have been certified in the use and maintenance of the air

driver.c. You have not been certified in adjusting torque.d. This would cause disruption on the line and interfere with

another’s work.e. This is an inappropriate response.

2. Correct answer: b

Why not?a. You are not qualified to determine this.c. Outside temperature will not affect your driver.d. All the torque drivers use the same grade oil.e. Outside humidity will not affect your driver.

3. Correct answer: c

Why not?a. If there was a leak in the hose, other drivers would also be affected.b. You are not empowered to do anything with the compressor.d. You cannot ignore the problem.e. Cooling the workstation will not affect the driver.

LESSON 2

46 • Applied Technology

4. Correct answer: b

Why not?a. The relative humidity will not affect the torque on your air driver.c. The number of workstations off the main supply is within

specifications.d. The hoses are made of fixed piping.e. A manual tool, like a screwdriver, would not give you appropriate

torque in a consistent manner and would slow down production.

5. Correct answer: c

Why not?a. Adding a fan will not affect the driver.b. This would disrupt others and close another tool station down.d. You are not empowered to adjust the psi.e. You will be unable to achieve the correct torque with a hand driver.

6. Correct answer: c

Why not?a. This would be an inappropriate action.b. This would be an inappropriate action.d. A visual inspection will not allow you to determine if the problem

has been corrected.e. Another fan will not solve the problem.

LESSON 2

Applied Technology • 47

ELECTRICITY

I will present an overview of electricity so that youcan briefly review basics discussed in previous levels.

Electricity is the continuous flow of electrons, orcurrent, from one atom to another. No electron flowwill occur unless there is a pathway over which theelectrons can move. This flow is similar to a water

system, where pipes or hoses move water fromstorage tanks to where it is needed. In electricalwiring, the pathway through which electrical

current flows is called a circuit. A simple circuitconsists of a power source, conductors, load,and a device for controlling current. Each is

described below.

In buildings, the power source could be theelectrical generating stations that pump electricity intoresidential and commercial buildings. However, othercommon sources of electrical power include smallgenerators and batteries.

Conductors, or wiring, provide a path for thecurrent so that it can travel from one point to another.

A load is a device through which electricity produceswork. For example, a lamp is a load that, when pluggedin and turned on, produces light. Other examples ofloads include heaters, electric motors, and televisions.

LESSON 3

48 • Applied Technology

Switches (on-off switches) control when electricalcurrent flows through circuits. Fuses and circuitbreakers are protective devices that control current bypreventing too much current from flowing in thecircuit, which would damage equipment. When anexcessive amount of electricity passes through them,fuses and circuit breakers “blow” or “trip” to stop theflow of electricity through the circuit.

In a circuit, resistance lowers the amount ofelectrical energy available to do work. Both wires andload affect resistance. It might be helpful to think of asimilar situation with a hose that is connected to twosprinklers. As water passes through a hose, turns orkinks in the pathway cause friction (which is resistance)that results in a slower flow. In addition, when some ofthe water is diverted to the first sprinkler (which is aload), less water is available for use in the secondsprinkler.

There are two ways or methods of having currentflow. Direct current flows in one direction. In mostcases, direct current is provided to equipment bybatteries (flashlights and portable radios). Alternatingcurrent flows in one direction, then reverses to the otherdirection. Alternating current is provided to equipmentthrough electrical substations in buildings. In theUnited States, common household current reversesitself 60 times per second. This results in 120v 60 cycleAC. The international reference for cycles is defined inhertz (one hertz = 1 cycle per second).

LESSON 3

Applied Technology • 49

Measurement of electric current

The rate at which electricity flows is calledamperage. It is measured in amperes. A 100-watt bulbrequires a current of approximately 1 ampere to makeit light up completely. Current flow is measured withan ammeter. Most electrically powered equipmentindicate the amount of current needed to operate itproperly.

Measurement of electrical pressure

Pressure is applied to electrons to force them to movethrough a conductor and around a circuit. This pressureis measured in volts. The pressure, or voltage, isavailable in wiring circuits all of the time … whetheror not electrical equipment is being used. Voltage ismeasured with a voltmeter.

Calculation of power

The amount of power derived from an electricaldevice or system is its wattage. In other words, it is theproduct obtained from electrical energy; it is the powerthat we put into use. For example, the electric companysells electrical energy. Electrical energy or power ismeasured in watts and can be calculated as follows:

For direct-current circuits:volts ××××× amperes = watts

For alternating-current circuits:volts ××××× amperes ××××× power factor = watts

___________________________________________________________________NOTE: Power factors range from 0-1. Large equipment (an electric heater) may have apower factor as high as 1; small equipment (a small motor) may have a power factor aslow as .25.

LESSON 3

Tip: Make note of thesemeasurements.

50 • Applied Technology

Ohm’s law

Ohm’s law is a simple formula used to describe therelationship between current (flow), voltage (pressure),and resistance of an electrical circuit. Each componentinteracts to affect the operation of a circuit. In otherwords, because voltage pushes current through aresistance, a change in any of the components will resultin a change in the others. The following three equationsare Ohm’s law rearranged to solve for each of thequantities:

Current = Voltage ÷ Resistance I = E/Ramps = volts ÷ ohms An increase in voltage causes an increase in

electrical current flow. An increase in circuitresistance causes a decrease in electricalcurrent flow.

Voltage = Current ××××× Resistance E = I ××××× Rvolts = amps × ohms An increase in current causes an increase in

voltage. An increase in resistance causes anincrease in voltage.

Resistance = Voltage ÷ Current R = E/Iohms = volts ÷ amps

LESSON 3

Applied Technology • 51

Generalizations that can be made about electricity:

• The longer the wire, the greater the resistance; the thinner the wire, the greaterthe resistance.

• An increase in temperature of a wire causes an increase in resistance.

• An ordinary electrical cord has two wires; one for the flow of current from thepower source and the other for the return or ground.

• The voltage (pressure) and current (flow of electricity) directly affect how muchpower is available to do work. Less energy source or lower flow will result in lesselectrical power being produced.

• A series circuit has only one path for the flow of current. In a series circuit,objects are placed one after another and the current flows through each of themin succession. The current is the same throughout, however, and the voltage isdivided among the objects in the circuit.

• In a parallel circuit, there are 2 or more paths, or branches, for the flow ofcurrent. The current will divide and flow through each of the pathssimultaneously. Every branch has the same voltage and - if the appliances are allthe same - will have the same amount of current. The total circuit resistance isless than any one branch.

• When batteries are connected in a series, the current is the same; the totalvoltage is the sum of the voltage of each battery. The terminals are connected+, -, +, -, and so on.

• When batteries are connected in parallel, the total current is the sum of thecurrents in each battery; the total voltage is the same as that of one cell. Theterminals are connected +, +, +, and -, -, -.

LESSON 3

52 • Applied Technology

Diagnostic equipment

There are many different types of test equipmentwhich can be used to troubleshoot electrical circuits.The most common testers include light probes,voltmeters, ohmmeters, ammeters, and oscilloscopes.

A handy basic tester is called a multimeter. This is arelatively inexpensive meter that can be used to measureresistance (ohms), voltage (AC or DC), or current(amps). They are available with an analog or digitalreadout.

To familiarize you with the use of the multimeter,we will discuss the analog readout version. The digitalmeter uses the same principles.

For the discussion, refer to Diagram B whichrepresents the face of a meter.

Let’s look a little closer at the meter’s operation. Allreadings are taken using the test probes. The probesare color-coded. The black probe is for the commonground/negative connection. The red probe is for thevoltage/positive connection. The probes are insertedin the meter.

It is imperative that the leads be in the rightreceptacles. Many test readings require the correctpolarity to obtain an accurate reading. The correctpolarity is for the black probe to be placed at the morenegative point of the circuit or component being testedand the red probe placed at the more positive point.Testing a circuit with power on and with the test probepolarity reversed will result in erroneous readings andwill likely damage the meter.

LESSON 3

Applied Technology • 53

It is extremely important when making the tests onactive circuits (voltage present), that the probes are heldonly by the plastic insulators and that you do notcontact the metal tips. Serious electrical shock or deathcould result.

The face of the meter displays several scales. Thescales include ohms, DC volts, and AC volts. The scaleused is determined by the selector switch setting.

It is important to note that while the multimetertester will measure current, the meter’s capacity forreading amperage is very low. The amperage capacityis normally in milliamps and only useful introubleshooting integrated and transistorized circuitry.Power circuits, household circuits, and the large currentapplications require an ammeter that has a largercapacity. Attempting to read a high current on astandard multimeter will damage the meter.

Diagram B

LESSON 3

54 • Applied Technology

Ohms/Resistance

Resistance, measured in ohms, is taken with thepower supply to the circuit or components turned OFF.

The first step is to place the function selector switchin the desired position. Refer to Diagram C.

Ohms/Resistance R×1 R×10 R×1K

Look at the scale labeled ohms and compare it tothe function selection. The needle reading at the levelof 5 on the scale with the selector on R×1 equals 5×1or 5 ohms. The needle reading at the level of 5 withthe selector on R×10 equals 5×10 or 50 ohms. Thesame reading with the selector on R×1K equals 5×1000or 5K ohms.

Prior to making a resistance measurement, the metermust be “zeroed.” This is accomplished by selecting thedesired meter range (R×1, etc.). The metal tips of theprobes are pressed together which creates a direct short.This means no resistance or “zero ohms.” While holdingthe tips together, the ohms adjustment is turned untilthe needle reads exactly zero ohms. Now, you are readyto make your resistance reading.

Just a couple of more tips … for the most accuratereadings, you should select the lowest possible meterrange (R1) that does not result in the meter fullydeflecting to the highest end of the scale. If the meterreads full deflection, move to the next higher scale andtry again. Also, you should make the “zero ohms”adjustment each time you change scales.

LESSON 3

Applied Technology • 55

Resistance measurements are taken by placing themeter in parallel with the measured circuit orcomponent.

LESSON 3

Diagram C

56 • Applied Technology

Voltage

The measurement of DC (direct current) and AC(alternating current) voltages are very similar. Propernegative and positive polarity are critical for DCmeasurements. In some AC applications, polarity is notimportant; however, it is good to follow proper polarityrules where possible.

Step one is to select the type of voltage to be read(AC or DC) and the proper range on the functionselector. For unknown voltages, the highest rangeshould be selected. Locate the proper readout scale onthe meter and calculate the reading based on the rangeselected. If the resulting reading fits in a lower scale,move to that scale for a more accurate reading.

For voltage readings, the meter does not need to be“zeroed.”

For voltage readings, the meter is placed in parallelwith the measured circuit or component. Refer toDiagram C.

LESSON 3

Applied Technology • 57

Current/Amps

Some multimeters will measure current, howeverthe amperage must be very low. Following the samesteps as outlined in resistance and voltagemeasurements, ensure the probes are in the propermeter receptacles. Proper polarity is critical. Select thecurrent function and desired range. Always select thehighest range first and move to lower ranges asappropriate.

Unlike the resistance and voltage measurements,the meter must be placed in series with the test circuit.This means the circuit must be broken and the meterattached so the current will flow directly through themeter.

LESSON 3

Series

58 • Applied Technology

,SAFETY TIP

When working with live circuits, clip the probes tothe desired test points before applying power andremove the power before removing the probes. If thisis not possible, clip one probe to a desired test point;hold the insulated portion of the remaining probe withone hand and place the metal tip at the second testpoint while keeping your other hand in your pocket.This reduces the potential of your hands touching itemsthat could complete an electrical path through yourbody, thereby reducing the potential for accidentalelectrical shock.

Standing on a rubber mat also reduces the possibilityof completing the circuit through your feet.

Now that we have reviewed the basics of electricity,it’s time to try a couple of problems relating to it.

LESSON 3

Applied Technology • 59

EXERCISE – USING A VOLTMETER

Instructions: In the following exercises, assume the batteries are 1.5 volts. Turn on yourvoltmeter and set it to the 5-volt DC scale.

1. Using a battery, touch the black meter lead to the negative terminaland the red meter lead to the positive terminal. What does the meterread?

____________________________________________________________

2. Reverse the meter leads. What does the meter indicate?

____________________________________________________________

3. In this circuit, a jumper lead connects two batteries in series. Whatdoes the meter read?

____________________________________________________________

LESSON 3

60 • Applied Technology

4. Reverse the polarity of one of the batteries in question 3. What doesthe meter read?

____________________________________________________________

5. With one battery and one light bulb in a circuit, what is the voltageacross the light bulb? ____________________________

LESSON 3

Applied Technology • 61

LESSON 3

6. With two light bulbs in series with the battery, what is the voltageacross each individual light bulb?___________________________

62 • Applied Technology

LESSON 3

ANSWERS TO EXERCISE

1. Using a battery, touch the black meter lead to the negative terminaland the red meter lead to the positive terminal. What does the meterread?

Answer: You would read 1.5 volts directly across the terminals.

2. Reverse the meter leads. What does the meter indicate?

Answer: The meter attempts to read negative and pegs the needle tothe negative side as you now have improper polarity. Largevoltage readings with improper polarity will damage the meter.

3. In this circuit, a jumper lead connects two batteries in series. Whatdoes the meter read?

Answer: With batteries attached in series with proper polarity, you willread the sum of the batteries’ voltage.(2)1.5 volt batteries = 3.0 volts

4. Reverse the polarity of one of the batteries in question 3. What doesthe meter read?

Answer: The meter reads “0.” The batteries are connected withimproper polarity. In order for voltage to be present, you musthave a positive and negative terminal. In this case, the leadsare attached to two positive terminals.

Applied Technology • 63

5. With one battery and one light bulb in a circuit, what is the voltageacross the light bulb?

Answer: 1.5 volts — Since you have only one component in the circuit,the entire voltage of the battery is applied to that component.

6. With two light bulbs in series with the battery, what is the voltageacross each individual light bulb?

Answer: .75 volts — In a series circuit, the applied voltage is dividedamong all the components in the circuit, depending on theirresistance. With bulbs of equal resistance, each would have.75 volts dropped for a total of 1.5 volts.

LESSON 3

64 • Applied Technology

LESSON 3

Pop Quiz:A circuit that has more than one path is calleda _________ circuit.

Applied Technology • 65

EXERCISE – CALCULATING VOLTAGE

Instructions: Refer to the circuit shown below and use Ohm’s law to answer the questions.

1. What is the total resistance in this circuit?

____________________________________________________________

2. What is the current in this circuit?

____________________________________________________________

3. What is the current through the 10Ω resistor and the 15Ω resistor?

____________________________________________________________

4. What is the voltage drop across the 10Ω resistor?

____________________________________________________________

5. What is the voltage drop across the 15Ω resistor?

____________________________________________________________

LESSON 3

66 • Applied Technology

ANSWERS TO EXERCISE

1. What is the total resistance in this circuit?

Answer: 25ΩIn a series circuit, total resistance equals the sum of theresistance of all components.

2. What is the current in this circuit?

Answer: .24 amps.24 amps = 6 volts divided by 25Ω (I = E/R)

3. What is the current through the 10Ω resistor and the 15Ω resistor?

Answer: .24 amps eachIn a series circuit, the total amps travel through eachcomponent.

4. What is the voltage drop across the 10Ω resistor?

Answer: 2.4 volts2.4 volts = .24 amps × 10Ω (E = I × R)

5. What is the voltage drop across the 15Ω resistor?

Answer: 3.6 volts3.6 volts = .24 amps × 15Ω (E = I × R)

How did you do on that one? If you had problems,go back, review, and try again. Remember, you areworking at your own pace, so take extra time to studyand review if you need.

LESSON 3

Applied Technology • 67

EXERCISE – CIRCUIT PREDICTIONS

Instructions: Study the following diagram and answer the related questions.

1. Predict what will happen when one bulb is removed from the socket.

Prediction: ______________________________________________

2. Predict what will happen when one bulb is removed from the circuitand the circuit is reconnected.

Prediction: ______________________________________________

3. Predict what will happen when the fuse is removed and the circuitis reconnected.

Prediction: ______________________________________________

LESSON 3

68 • Applied Technology

4. Predict what will happen when the fuse is replaced with a burned-out fuse.

Prediction: ______________________________________________

5. Predict what will happen when the resistor is removed and the circuitis reconnected.

Prediction: ______________________________________________

6. Predict what will happen if the operational resistor is replaced witha burned-out resistor.

Prediction: ______________________________________________

In Diagram D, a circuit has been built similar to the diagram used earlier in this exercise.The circuit is the same, but a parallel branch with one bulb has been added.

LESSON 3

Diagram D

Applied Technology • 69

7. Predict what will happen if Bulb 4 is removed from the socket.

Prediction: ______________________________________________

8. Predict what will happen if Bulb 1 is removed from the socket.

Prediction: ______________________________________________

9. In the space provided, summarize what you learned about electricalsystems from the activities included in the exercise. If you are unsureabout the accuracy of your answers for questions 1–8, review theanswers to the exercise before completing your summary.

____________________________________________________________

____________________________________________________________

____________________________________________________________

____________________________________________________________

____________________________________________________________

____________________________________________________________

LESSON 3

70 • Applied Technology

ANSWERS TO EXERCISE

1. Predict what will happen when one bulb is removed from the socket.

Answer: All of the bulbs go out. There is only one path for current andthat path has been broken.

2. Predict what will happen when one bulb is removed from the circuitand the circuit is reconnected.

Answer: The two bulbs burn brighter. The circuit voltage divides acrossall resistance in a series. The removal of one bulb creates ahigher voltage applied to the remaining components.

3. Predict what will happen when the fuse is removed and the circuitis reconnected.

Answer: There is no change. The lights burn. The fuse has no electricalfunction beyond being a circuit safety function.

4. Predict what will happen when the fuse is replaced with a burned-out fuse.

Answer: No current flows and nothing lights. The circuit is open.

5. Predict what will happen when the resistor is removed and the circuitis reconnected.

Answer: Lights will burn brighter depending on the size of the resistor.

LESSON 3

Applied Technology • 71

6. Predict what will happen if the operational resistor is replaced witha burned-out resistor.

Answer: No current flows and nothing lights. The circuit is open.

7. Predict what will happen if Bulb 4 is removed from the socket.

Answer: Bulbs 1, 2, and 3 burn.

8. What happens when Bulb 1 is removed from the socket?

Answer: Bulbs 2 and 3 go out. Bulb 4 burns.

9. In the space provided, summarize what you learned about electricalsystems from the activities included in the exercise. If you are unsureabout the accuracy of your answers for questions 1–8, review theanswers to exercise before completing your summary.

Answers will vary.

LESSON 3

How well did you predict?

72 • Applied Technology

EXERCISE – MACHINE INDICATOR LIGHTS

Instructions: Read the scenario and refer to the following diagram to answer the questionsabout this problem.

Scenario

You are the maintenance person at a plant. Part of yourresponsibility is to check the indicator lights on all machinery.For safety reasons, indicator lights must be lit while themachines are running. As you walk through the plant, younotice that a machine is running, but its indicator light is out.You take voltage measurements across the variouscomponents V1, V2, V3, and V4, as indicated on the followingschematic. It is your responsibility to determine the problemand correct it.

LESSON 3

Applied Technology • 73

Identify the problem

1. What is your assignment?

a. to determine if the power supply is goodb. to determine the size of the lampc. to determine the size of the motord. to determine the reason why the indicator light is out although

the machine is one. to determine the length of the wire from the motor to the lamp

Define the problem

2. What do you need to consider to solve this problem?

a. the wattage of the lampb. which circuit is involved with the lamp being outc. the size of the power supplyd. the size of the fusee. the gauge of the wire

Explore alternatives

3. Does your voltage reading correspond with what should behappening if there is current flowing through each device?

a. Zero voltage across the fuse indicates the fuse is the problem.b. Twelve volts across the resistor indicates the resistor is open.c. Zero volts across the lamp indicates the light is bad.d. The fuse is bad because there is no current.e. The DC supply in not functioning.

LESSON 3

74 • Applied Technology

IDEALRemember to use …

LESSON 3

… for solving problems.

Applied Technology • 75

Act on a plan

4. What is the appropriate action to correct the problem?

a. Replace the resistor.b. Replace the lamp.c. Replace the fuse.d. Buy a new motor.e. Buy a new power supply.

Look at the result

5. Does the indicator light work?

a. yesb. no

LESSON 3

76 • Applied Technology

ANSWERS TO EXERCISE

1. What is your assignment?

Answer: d. to determine the reason why the indicator light is outalthough the machine is on

2. What do you need to consider to solve this problem?

Answer: b. which circuit is involved with the lamp being out

3. Does your voltage reading correspond with what should behappening if there is current flowing through each device?

Answer: b. Twelve volts across the resistor indicates the resistor isopen.

4. What is the appropriate action to correct the problem?

Answer: a. Replace the resistor.

5. Does the indicator light work?

Answer: a. yes

LESSON 3

Applied Technology • 77

MECHANICS

This is our last lesson in this level of AppliedTechnology. How have you been doing so far? Do youunderstand the basic idea of the lessons? I sure hopeso. If not, when you finish, go back and review all thatyou need to. I don’t mind waiting!

First, let’s review basic mechanics just as we havedone in all the other lessons. This will include somegeneralizations that can be made about mechanics aswell as a description of the simple machines that wehave discussed earlier.

This is mainly for the benefit of students who arejust starting this course at this level, but for all of youwho have been with me from the beginning, it will bea good review.A machine is something that does work.

LESSON 4

What basic machine principle am I using? You willfind the answer as you review the basics.

78 • Applied Technology

Work is done when a force causes an object to move.

Simple machines (gears, pulleys, inclined planes,levers, wheel and axle), when used together, make upcompound (or complex) machines.

Examples of compound machines include a bicycle,a rod and reel, a typewriter, a can opener, scissors, ahand drill, a car, a weight machine, and a treadmill.

Gears• The force that is applied to a driver gear is

transferred to a driven gear.• When two gears of different sizes are meshed

together, the smaller gear turns faster (morerotations per minute) than the larger gear.

• Gears that are meshed together move in oppositedirections.

• The direction and speed of the driver geardetermines the speed and direction of gears thatare meshed with it.

LESSON 4

Applied Technology • 79

Pulleys• A pulley is a wheel with a rope, belt, or chain around

it.• Pulleys change the direction of movement and make

work easier.• Fixed pulleys change the direction that something

is moved; they do not make work easier.• Movable pulleys change the direction that

something is moved and make work easier.• The more pulleys in the system, the easier it is to

do work (pull or lift an object).• The more pulleys involved in a system, the greater

distance must be pulled, but the easier it is to dowork.

• The thinner the windlass (winch), the easier it is toturn.

• In two different sets of pulleys, if the wheels areconnected by a shaft and the two wheels on onepulley are the same size as the two wheels on theother pulley, they will both turn at the same speed.

• Common pulley applications include crankshafts,sailboats, boat lifts, and window blinds.

Inclined Planes• An inclined plane is a slanted surface that is used to

raise or lower heavy objects from one position toanother.

• Inclined planes help reduce the amount of forceneeded to do a given amount of work, but requiregreater distance.

• The steeper the plane, the more difficult the work.• *Wedges are two back-to-back inclined planes.• Common inclined plane applications include

*screws, bolts, drill bits, clamps, car jacks, loadingramps, and screw-on bottle tops.

LESSON 4

*Some textbooks refer to the wedge and screw as basic simple machines.

80 • Applied Technology

Levers• A lever is a bar or rod that is free to move or turn

on a fulcrum.• A lever multiplies force, but some distance must be

given up.• The shorter the effort arm, the less force is attained

and the greater distance is attained.• The longer the effort arm, the more force is attained

and the less distance is attained.• Examples of levers include scissors, a broom, a claw

hammer, a nutcracker, a mop, tongs, a crowbar, acan opener, tweezers, a baseball bat, boat oars, anda car jack handle.

Wheel and Axle• A wheel and axle is like a spinning lever (an ice

cream machine crank).• The center of the axle is the fulcrum.• The wheel is larger than the axle; for one rotation,

a point on the edge of the wheel travels a greaterdistance than a point on the axle. While the workdone by the axle and the wheel are the same, thegreater distance traveled of the point on the edge ofthe wheel yields a smaller force at the edge of thewheel versus the edge of the axle.

• Common wheel and axle applications include ascrewdriver, roller skates, a water-faucet handle, abicycle pedal, a can opener, and a car steering wheel.

LESSON 4

Applied Technology • 81

Internal Combustion EnginesInternal combustion engines are examples of

compound/complex machines. Many differentapplications of simple machines occur in the engine.Internal combustion engines take input energies andconvert them to an energy output to do work. Examplesof common uses of the engine include the automobile,lawn mower, gasoline-powered trimmers, gasoline-powered blowers, and gasoline-powered generators.

For our discussion, we will be featuring the four-cycle engine found on most lawn mowers. This is afairly simple engine; however, the same principles applyto more complex applications such as the automobile.

We will omit several components from ourillustrations to focus on the base operations. However,you should be aware that they exist and will be locatedon the engine. Most owner manuals will provideillustrations to help locate these components. Theyinclude the gas tank, fuel filter, air filter, oil filter, oilfill and crankcase, muffler, etc.

OK, let’s take a look at the logic behind the internalcombustion engine. We are going to cause a smallexplosion inside an airtight cylinder. The explosion iscreated by firing a spark plug in a mixture of gas andair. This explosion and the expanding gases will push apiston down in the cylinder. The piston is attached toa crankshaft which is turned as a result of the pistonmovement. The turning crankshaft provides the poweroutput to operate transmissions, pulleys, or otherdevices.

Let’s look closer at the four cycles or four strokes ofa four-cycle engine.

LESSON 4

82 • Applied Technology

1. Intake stroke – the intake valve is opened andthe downward movement of the piston drawsin an air-fuel mixture from the carburetor.

2. Compression stroke – as the camshaft rotates,the intake valve is closed. The piston is pushedback up the cylinder and compresses the air-fuelmixture.

LESSON 4

Applied Technology • 83

3. Power stroke – the spark plug ignites the fuelmixture, forcing the piston down. The pistonmovement turns the crankshaft.

4. Exhaust stroke – Momentum drives the pistonin an upward direction in the cylinder. Theexhaust valve is opened and the burned gasesare forced out.

LESSON 4

84 • Applied Technology

In order to provide output power to the crankshaft,this four-cycle process must be repeated very rapidly.Also, different functions within the process must occurin the right cycle and at exactly the right time.

The important timing considerations are the firingof the spark plug and the opening and closing of theintake and exhaust valves. For example, firing of thespark plug during the exhaust cycle would result in amisfire. Sufficient fuel would not be present fordetonation … any force developed would escapethrough the exhaust valve and force could not be exertedon the piston as it is its lower position.

Proper timing is achieved through the use of gearsand cams on a camshaft. A cam is an irregular lobemachined on a camshaft. The projection and otherportions of the cam are used to control the time andduration of repetitive operations in a machine.

Let’s look at the opening and closing of the intakeor exhaust valve under control of a cam.

LESSON 4

Applied Technology • 85

In Figure 1, the valve is closed as the lobe approachesthe valve operating link.

Figure 1

In Figure 2, the lobe has pushed the valve open.

Figure 2

In Figure 3, the lobe has moved past the valve and thespring on the operating link has closed the valve.

Figure 3

LESSON 4

86 • Applied Technology

Several cams with different shapes can be locatedon a single camshaft to control different functions atdifferent times of the cycle.

The cams controlling the intake and exhaust valveswould be on the same shaft and be similar in shape.However, the lobes would be spaced differently toactivate the valves independently at the proper time.

The camshaft is driven by the crankshaft. Thecrankshaft will make two revolutions as the engine goesthrough its four cycles while the camshaft will gothrough one revolution. This is accomplished bygearing from the crankshaft.

The firing of the spark plug is also controlled bycams. A high voltage charge is developed through theignition coil. This voltage is applied to the spark plugby the opening and closing of ignition points that actas a switch. A cam operates the points to open andclose them at the correct time.

LESSON 4

Applied Technology • 87

OK, let’s review the sequence of operation.

1. On the intake stroke, the intake valve is opened by the cam on the camshaftand the downward movement of the piston draws in the air-fuel mixture fromthe carburetor. The cam operating the exhaust valve is on the low side of thecam, so the exhaust valve is closed.

2. On the compression stroke, the cam operating the intake valve has moved tothe low side of the cam and the intake valve is closed. The cam controlling theexhaust valve is still on the low side and the exhaust valve remains closed. Thepiston moves back up the cylinder and compresses the air-fuel mixture.

3. On the power stroke, both intake and exhaust valve control cams are at thelow point and both valves remain closed. The ignition points close, driven bythe ignition cam. A high voltage is developed through the ignition coil and isapplied to the spark plug. A spark jumps across the gap of the spark plug,igniting the fuel mixture. The piston is forced down rapidly by the explosionand expanding gases. The connecting rod between the piston and crankshaftcauses the crankshaft to turn.

4. On the exhaust stroke, the exhaust valve is opened by the cam. The intakevalve remains closed. Momentum drives the piston upward in the cylinderand the burned gases are forced out the exhaust valve.

5. Work output occurs through the turning of the crankshaft.

LESSON 4

88 • Applied Technology

As I indicated earlier, the principles of operationfor an automobile engine are the same as what youhave just learned. An automobile engine may have four,six, eight, or twelve cylinders functioning the same asthe single cylinder we have been studying. All thecylinders will be connected to and driving the samecrankshaft. The linkage and timing of the cylinderoperation is designed to drive the crankshaft withmaximum efficiency and power.

The automobile engine will obviously have moresophisticated controls and timing devices than a singlecylinder engine. These include computer chips andelectronic devices to control fuel mixtures, pollutioncontrol components, ignition, accessories, and manyother marvels of today’s automobile.

Another major difference in engines is the coolingsystems. Smaller engines are air cooled with heat beingdissipated through fins on the engine. The multiplecylinders of larger engines create more heat and requireadditional cooling. Most larger engines like those usedin automobiles are water cooled.

Water-cooled engines circulate a mixture of waterand antifreeze/coolant through the engine block undercontrol of a thermostat. The coolant mixture passesthrough the radiator where it is cooled by the radiatorfan and outside air. The thermostat senses the enginetemperature and allows enough coolant to pass tomaintain the optimum operating temperature.

LESSON 4

Applied Technology • 89

+

LESSON 4

In the summer, after the engine has warmed up,the thermostat will normally be wide-open and providemaximum cooling. During the winter, the thermostatmay be only partially open to allow the engine tomaintain the best operating temperature.

Our first problem will be focusing on identifyinglogical cause for the mechanical failure of an enginewhen given a specific set of circumstances. This willhelp you identify the best course of action to fix amechanical problem and to examine the effectivenessof your work.

Many times it is difficult to see the relationship ofone component to an entire mechanical system … inother words to see “the big picture.” Our first problemwill help you understand the effect that valves have onan engine system. You will be asked questions thatrequire you to use logic.

90 • Applied Technology

EXERCISE – LAWN MOWER START-UP

Instructions: Read the scenario and study the following diagram to answer the questionsabout this problem.

Scenario

You work at a local lawn mower repair shop. One day, acustomer brings in her lawn mower and explains that it willnot start. She has already checked the fluid levels anddetermined that the spark plug is in working order. When shepulls the start cord, however, it seems to pull too easily, asthough there is very little resistance. What can be done torestore the lawn mower to working order? Use the diagramas needed.

LESSON 4

Applied Technology • 91

Identify the problem

1. What is the problem?

a. It is too difficult for the customer to pull the cord properly.b. The lawn mower will not start.c. The customer wants to be able to start her lawn mower by using

a key instead of a start cord.d. The lawn mower brand is not the brand the customer is used to

operating.e. The length of the start cord must be at least 15 feet.

2. Why doesn’t the lawn mower start?

a. It has no fuel to power the engine.b. It has no oil to lubricate the engine.c. There is little resistance on the start cord.d. The spark plug needs to be replaced.e. The customer lacks the strength to make the engine turn over

fast enough when she pulls the start cord.

Define the problem

3. Why doesn’t the start cord have enough resistance?

a. The start cord is broken in two.b. The engine has too much oil.c. The start cord has become frayed from wear and tear.d. There is no engine compression.e. There is no engine lubricant.

LESSON 4

92 • Applied Technology

Explore alternatives

4. If the engine’s valves were stuck open, the effect would be as follows:

a. The engine would run but not as efficiently.b. The piston would not move up and down.c. The start cord would pull with little resistance; the crankshaft

would move the piston up and down, but there would be nocompression; and the engine would not start.

d. The start cord would pull with great resistance; the crankshaftwould move the piston up and down, and the engine would havecompression and start.

e. The open valves would result in higher engine performance.

Act on a plan

5. Assuming there is nothing wrong with the starting-cord mechanism,what would be your first step in determining why the start cord haslittle resistance?

a. Check the tightness of all engine bolts.b. Dismantle the lawn mower engine to explore possible problems.c. Do an engine compression test.d. Make sure the lawn mower has the blade attached.e. Check to be sure the throttle is in the “run” position.

LESSON 4

Applied Technology • 93

6. Which of the following causes would be the MOST accurate reasonthe engine has no compression?

a. The intake and exhaust valves are closed while the piston istraveling upward during the compression stroke.

b. The intake valve is open and the exhaust valve is closed whilethe piston is traveling upward during the compression stroke.

c. There is corrosion on the outside of the engine.d. The intake valve is open and the exhaust valve is closed while

the piston is traveling downward during the intake stroke.e. The engine appears old and outdated.

Look at the result

7. After repairing stuck valves and reassembling the engine, whatshould you do next?

a. Immediately call the customer and tell her the lawn mower is fixed.b. Ask your boss for a pay raise for a job well done.c. Using a tension indicator, test the start cord to make sure it has

proper resistance.d. Test mow a patch of lawn at least 150´ by 150´.e. Test the engine compression, then start the mower to make sure

it works.

LESSON 4

94 • Applied Technology

ANSWERS TO EXERCISE

1. What is the problem?

Answer: b. The lawn mower will not start.

2. Why doesn’t the lawn mower start?

Answer: c. There is little resistance on the start cord.

3. Why doesn’t the start cord have enough resistance?

Answer: d. There is no engine compression.

4. If the engine’s valves were stuck open, the effect would be as follows:

Answer: c. The start cord would pull with little resistance; thecrankshaft would move the piston up and down, but therewould be no compression; and the engine would not start.

5. Assuming there is nothing wrong with the starting-cord mechanism,what would be your first step in determining why the start cord haslittle resistance?

Answer: c. Do an engine compression test.

LESSON 4

Applied Technology • 95

6. Which of the following causes would be the MOST accurate reasonthe engine has no compression?

Answer: b. The intake valve is open and the exhaust valve is closedwhile the piston is traveling upward during the compressionstroke.

7. After repairing stuck valves and reassembling the engine, whatshould you do next?

Answer: e. Test the engine compression, then start the mower to makesure it works.

LESSON 4

IDEAL

96 • Applied Technology

Spatial visualization involves the ability to

manipulate and mentally rotate two-dimensional andthree-dimensional objects. Spatial orientation involvesthe ability to perceive the elements in a pattern, tocompare patterns, and to grasp changing orientationin space. Whew, that sounds like a lot, doesn’t it? Butreally it’s not that difficult when you break it down tosomething with which you are familiar.

Take a child’s square building block, for example.When you hold it in your hand, you can rotate it inany direction, and you can see the three-dimensionalityof it. The only difference is that you must “see” this inyour mind when you are looking at a square block ona piece of paper (probably in two-dimensions).

Patterns and elements in a pattern are easilydemonstrated by a multicolored Rubic’s cube.Remember those? They were a huge fad a few yearsago. I think every household in America owned at leastone. I know I did!

See, you already know what I mean, don’t you?That’s because you have experience with them. Thesetypes of spatial skills are highly correlated with thesuccess you can achieve in a number of technical andprofessional employment situations. Spatial orientationis necessary to have a good sense of direction for taskssuch as reading schematics, diagrams, or even maps.

LESSON 4

Applied Technology • 97

<

Let’s complete a learning activity. See if you haveever had to do something like this. I’ll bet you have!

Thinking Activity

Suppose you are moving and you have to load a refrigerator, an emptydresser, and various boxes of miscellaneous items into a truck. Which wouldyou load first? Why?

Here’s what I would do. Compare your answer withmine.

I would load the refrigerator first, placing it againstthe wall near the cab of the truck. This places the weightforward of the rear axle. By placing the refrigerator inthe center of the wall, rather than against a left or rightwall, you will distribute the weight. This will preventthe vehicle from being difficult to handle.

Then, I would load the next biggest item, the emptydresser, in one of the back corners. I would fit the boxesand other miscellaneous items around the larger onesfor the best stability and least movement. Heavier boxesshould go on the opposite side of the dresser as muchas possible in order to balance the weight of the dresser.

Let’s try an exercise.

LESSON 4

98 • Applied Technology

EXERCISE – TRUCK LOADING

Instructions: Read the following scenario and answer the questions considering spatialorientation.

Scenario

You are an employee working in the shipping department onthe shipping dock. You are instructed to package a 3˝ x 6˝ x 2´part into a carton. It can go into the carton in any way. You areasked to find the carton that will result in the least wastedspace and to assemble it to pack the part. Cartons are stackedflat for storage. You are to add this box to the balance of theday’s production. You are then to load the truck. You have 10skids to load. They are all the same size, but they are differentweights. The skids will fill a truck when stacked 2 skids wideand 5 skids deep. A truck backs into the dock, ready to load.You must determine which size of box to use and the bestmethod for loading the truck.

Identify the problem

1. What is your assignment?

a. You must order cartons from the supplier.b. You are to direct customers to the back room.c. You are to package a part.d. You must find and assemble a carton, pack a part, and load a

truck.e. You must operate a forklift and train a helper in its use.

LESSON 4

Applied Technology • 99

Define the problem

2. What do you need to consider when completing your assignment?

a. You must assemble a carton to accommodate a 3˝ x 6˝ x 2´ partand send it abroad.

b. You need to determine the exact size of the carton, load a skid,and complete the shipping label.

c. You must locate and assemble a carton of the proper size, packthe part, and load a balanced truck.

d. You must load a truck for shipping.e. You must contact the shipping department before loading the

truck.

Explore alternatives

3. Which sequence of events will help you achieve your goal?

a. Change the 2´ to 24˝; locate the approximately sized box; assemblethe carton; pack the item in the carton; and add the box to thelast skid.

b. Select a carton; package the part; arrange the skids so that theweights are balanced on each side of the truck; and load the truck.

c. Change the 2´ to 24˝; give the part to a helper to pack; and loadthe truck.

d. Put all the dimensions in the same denomination; select a flatcarton that, when assembled, will accommodate the part;assemble the carton and pack the part; add the carton to theday’s production; and load the skids so that the weights arebalanced on each side of the truck.

e. Wrap the part in brown paper; and load the truck with the skids innumerical order.

LESSON 4

100 • Applied Technology

Act on a plan

4. From the choices below, select the MOST appropriately sized boxand assemble it by folding the creased lines.

a. Box A is the best box to use.b. Box B is the best box to use.c. Box C is the best box to use.d. Box D is the best box to use.e. None of the boxes is the right size.

LESSON 4

Applied Technology • 101

5. From the following select the MOST appropriately arrangedconfiguration to load into the truck.

a. Truckload A is best.b. Truckload B is best.c. Truckload C is best.d. Truckload D is best.e. None of the configurations will work.

Look at the result

6. Does the item fit into the box?

a. Yes, the part fits into the box.b. No, the part is too big for the box.

LESSON 4

102 • Applied Technology

7. How can you determine whether the truck is loaded in a balancedmanner?

a. Load only the center of the truck, and leave the sides empty toensure balance.

b. Visually inspect the truck to see whether it leans to one side.c. No check is necessary.d. Drive the truck to a weigh station on the highway and weigh the

truck.e. Add up the total weights on each side of the truck.

IDEAL

LESSON 4

Applied Technology • 103

ANSWERS TO EXERCISE

1. What is your assignment?

Answer: d. You must find and assemble a carton, pack a part, andload a truck.

2. What do you need to consider when completing your assignment?

Answer: c. You must locate and assemble a carton of the proper size,pack the part, and load a balanced truck.

3. Which sequence of events will help you achieve your goal?

Answer: d. Put all the dimensions in the same denomination; select aflat carton that, when assembled, will accommodate thepart; assemble the carton and pack the part; add the cartonto the day’s production; and load the skids so that theweights are balanced on each side of the truck.

4. From the choices below, select the MOST appropriately sized boxand assemble it by folding the creased lines.

Answer: c. Box C is the best box to use.

5. From the following select the MOST appropriately arrangedconfiguration to load into the truck.

Answer: a. Truckload A is best.

LESSON 4

104 • Applied Technology

6. Does the item fit into the box?

Answer: a. Yes, the part fits into the box.

7. How can you determine whether the truck is loaded in a balancedmanner?

Answer: e. Add up the total weights on each side of the truck.

How did you do? If you had trouble, go back andreview to help reinforce the concept. Then, let’s moveon to our next problem.

The focus of this exercise is to help you to identifythe logical cause for mechanical failure of a conveyorsystem. It will also help you identify the course of actionto fix a mechanical problem and to examine theeffectiveness of your work.

As I mentioned before, sometimes it is difficult tosee the “big picture” of a system, or how one componentrelates to an entire mechanical system. Our nextproblem should help you gain an understanding of theeffect of temperature changes on the functioning of amechanical system.

LESSON 4

Applied Technology • 105

EXERCISE – PELLET-TRANSFER SYSTEM

Instructions: Read the following scenario and answer the related questions. There is nodiagram provided with this problem. This will give you an opportunity todraw a diagram that fits the description provided in this problem.

Scenario

A long pan is used to convey pellets a horizontal distance …from the silo to the hoppers. The pan vibrates to move thecontents. The vibration is set up by an attached vibratingshaker drive. This drive rotates weights in opposite directionsby gearing. Alignment of the gearing is held by keys inkeyways in the driveshaft. This driveshaft is driven by anelectric motor. The side of the conveyor has a “V” symbolmounted on it, which indicates the degree of horizontalvibration. The technician determines the degree of horizontalvibration by observing the visual overlap of the sides of the“V” while the conveyor is moving.

You are the technician responsible for making sure that thepellet-transfer system operates properly. You observe that theconveyor is vibrating, but it is not moving pellets. You needto determine why the contents are not being moved.

LESSON 4

106 • Applied Technology

Identify the problem

1. What is your assignment?

a. to clean the conveying panb. to check the drive motor loadc. to find out why the material does not conveyd. to replace the vibrating conveyor with a belte. to replace the conveying pan

Define the problem

2. How can the problem be described?

a. The motor does not run.b. The conveyor does not vibrate.c. The motor runs too fast.d. The conveyor vibrates, but the material does not move.e. The motor runs too slowly.

Explore alternatives

3. What are the possible causes?

a. The conveyor is vibrating improperly.b. The drive motor is burned up.c. The conveyor is too long.d. The conveyor is too short.e. The drive motor stopped.

LESSON 4

Applied Technology • 107

Act on a plan

4. What would you do to check to see if the vibration has beendisrupted?

a. Check the motor load.b. Check the moisture of the contents.c. Measure the length of the conveyor.d. Check the shaker drive’s mounting; verify that both weights are

rotating in opposite directions.e. Check the motor rpm.

Look at the result

5. You found one weight rotating; the other weight sheared its key andis not moving. What would you do?

a. Replace the motor.b. Lubricate the bearings; spin the driveshaft.c. Dry the pan contents; replace the pan.d. Replace the sheared key; check vibration and material movement.e. Check the vibration of the pan.

LESSON 4

IDEAL

108 • Applied Technology

ANSWERS TO EXERCISE

1. What is your assignment?

Answer: c. to find out why the material does not convey

2. How can the problem be described?

Answer: d. The conveyor vibrates, but the material does not move.

3. What are the possible causes?

Answer: a. The conveyor is vibrating improperly.

4. What would you do to check to see if the vibration has beendisrupted?

Answer: d. Check the shaker drive’s mounting; verify that both weightsare rotating in opposite directions.

5. You found one weight rotating; the other weight sheared its key andis not moving. What would you do?

Answer: d. Replace the sheared key; check vibration and materialmovement.

LESSON 4

Applied Technology • 109

Well, that’s it for Level 5 except for the Posttest, ofcourse! When you are ready and you have reviewed thematerial, go ahead and try the test. I know you’ll dowell!

LESSON 5

Don’t peek at the answers!

110 • Applied Technology

EXERCISE – POSTTEST

Instructions: Using the knowledge you gained in this level of Applied Technology, answerthe following questions.

1. Bernoulli’s principle states that the faster the flow of air or fluid, the(lower, higher) the pressure. (Circle the correct word.)

2. All matter is made up of particles that are in _____________________.

3. Newton’s third law of motion states that for every action there is

____________________________________________________________.

4. The study of heat is called:

a. hydraulicsb. thermodynamicsc. pneumatics

5. Name two devices that are components of a heating and coolingsystem.

____________________________________________________________

____________________________________________________________

6. What is a diffuser in relation to heating and cooling systems?

____________________________________________________________

POSTTEST

Applied Technology • 111

7. The total amount of air that determines the room temperature iscalled _________________________.

8. What is the purpose of the blower motor?

____________________________________________________________

____________________________________________________________

9. What is the purpose of the thermostat?

____________________________________________________________

____________________________________________________________

10. _____________________ is achieved when the temperature in allrooms are equal within one degree.

11. Besides protecting from electrical overload, what is another purposeof fuses and circuit breakers?

____________________________________________________________

____________________________________________________________

12. What is the purpose of vents in a piece of equipment?

____________________________________________________________

____________________________________________________________

POSTTEST

112 • Applied Technology

13. What does the fan in a computer do?

____________________________________________________________

____________________________________________________________

14. A fan moves the air around the room, but it does not actually coolthe air. True or False?

15. Heat always stays near the floor while cooler air rises to the ceiling.True or False?

16. Another word for pneumatic pressure is _______________.

17. Water or oil pressure is called _________________.

18. What benefit does compressed air provide?

____________________________________________________________

____________________________________________________________

19. A water flow system has ________________________ to control anddirect the flow of water.

POSTTEST

Applied Technology • 113

20. In a home water system, when you open all the faucets, what willhappen?

____________________________________________________________

____________________________________________________________

____________________________________________________________

21. Current that flows in only one direction is called:

a. alternating currentb. direct current

22. The rate at which current flows is called:

a. voltageb. amperagec. wattage

23. The amount of power derived from an electrical device is:

a. voltageb. amperagec. wattage

24. Pressure is applied to electrons to force them through a conductivematerial. This pressure is measured in:

a. voltsb. ampsc. watts

POSTTEST

114 • Applied Technology

POSTTEST

25. Name at least three simple machines.

____________________________________________________________

____________________________________________________________

____________________________________________________________

26. A ________________ machine is one that is made up of more thanone simple machine.

27. Give two examples of a compound or complex machine.

____________________________________________________________

____________________________________________________________

28. A ______________________ is something that does work.

29. A machine that consists of two inclined planes placed back to backis called a ____________________.

30. The center of an axle is called the:

a. leverb. hubc. fulcrum

Applied Technology • 115

POSTTEST

31. How would you load a truck with the following items: piano, dresser,four chairs, assorted boxes?

____________________________________________________________

____________________________________________________________

____________________________________________________________

32. Why would positioning be important in the previous question?

____________________________________________________________

____________________________________________________________

____________________________________________________________

33. ________________ ________________ involves the ability tomanipulate and mentally rotate two- and three-dimensionalobjects.

34. A window blind is an example of:

a. gearb. inclined planec. pulley

35. A wood screw is an example of:

a. gearb. leverc. inclined planed. wedge

116 • Applied Technology

ANSWERS TO EXERCISE

1. Bernoulli’s principle states that the faster the flow of air or fluid, the(lower, higher) the pressure.

Answer: lower

2. All matter is made up of particles that are in ________________.

Answer: constant motion

3. Newton’s third law of motion states that for every action there is______________________________.

Answer: an equal and opposite reaction

4. The study of heat is called:

Answer: b. thermodynamics

5. Name two devices that are components of a heating and coolingsystem.

Answer: Any two of the following:compressor shutoff valvediffusers filtersductwork blowercondenser evaporatorthermostat

POSTTEST

Applied Technology • 117

6. What is a diffuser in relation to heating and cooling systems?

Answer: A vent usually placed in the ceiling or flooring that allowsairflow adjustment.

7. The total amount of air that determines the room temperature iscalled _________________________.

Answer: volume

8. What is the purpose of the blower motor?

Answer: The blower motor forces the air into the ductwork.

9. What is the purpose of the thermostat?

Answer: The thermostat senses and controls the temperature in therooms.

10. _____________________ is achieved when the temperature in allrooms are equal within one degree.

Answer: Equilibrium

11. Besides protecting from electrical overload, what is another purposeof fuses and circuit breakers?

Answer: They stop the electrical current when the maximumtemperature on a device is exceeded, preventing overheatingof the device. This protects the individual components in thesystem.

POSTTEST

118 • Applied Technology

12. What is the purpose of vents in a piece of equipment?

Answer: ventilation, allowing air to move so that heat will notaccumulate

13. What does the fan in a computer do?

Answer: cools the components

14. A fan moves the air around the room, but it does not actually coolthe air. True or False?

Answer: true

15. Heat always stays near the floor while cooler air rises to the ceiling.True or False?

Answer: false

16. Another word for pneumatic pressure is _______________.

Answer: air pressure

17. Water or oil pressure is called _________________.

Answer: hydraulic pressure

18. What benefit does compressed air provide?

Answer: Compressed air allows a device to apply much more powerto do work.

POSTTEST

Applied Technology • 119

19. A water flow system has ________________________ to control anddirect the flow of water.

Answer: valves

20. In a home water system, when you open all the faucets, what willhappen?

Answer: Water pressure will be reduced at all the faucets.

21. Current that flows in only one direction is called:

Answer: b. direct current

22. The rate at which current flows is called:

Answer: b. amperage

23. The amount of power derived from an electrical device is:

Answer: c. wattage

24. Pressure is applied to electrons to force them through a conductivematerial. This pressure is measured in:

Answer: a. volts

POSTTEST

120 • Applied Technology

25. Name at least three simple machines.

Answer: Any three of the following:wheel and axlepulleygearinclined planelever

26. A ________________ machine is one that is made up of more thanone simple machine.

Answer: compound or complex

27. Give two examples of a compound or complex machine.

Answers: Any two of the following:bicycle cartypewriter weight machinescissors can openertreadmill hand drillrod and reel (numerous others)

28. A ______________________ is something that does work.

Answer: machine

29. A machine that consists of two inclined planes placed back to backis called a ____________________.

Answer: wedge

POSTTEST

Applied Technology • 121

30. The center of an axle is called the:

Answer: c. fulcrum

31. How would you load a truck with the following items: piano, dresser,four chairs, assorted boxes?

Answer: Put the piano in the center of the wall nearest the truck cab.Put the dresser in one of the back corners. Make sure thatthe heavier boxes are on the opposite side as the dresser.Put chairs against the side walls and assorted boxes aroundthe other furniture.

32. Why would positioning be important in the previous question?

Answer: The proper balance must be maintained to provide the bestequilibrium possible.

33. ________________ ________________ involves the ability tomanipulate and rotate mentally two- and three-dimensionalobjects.

Answer: Spatial visualization

34. A window blind is an example of:

Answer: c. pulley

35. A wood screw is an example of:

Answer: c. inclined plane

POSTTEST

122 • Applied Technology

Calculate your score counting the number of questions you answered correctly. If aproblem asked you to list several items or steps and you missed one or more, count thequestion as answered incorrectly. Divide the number of your correct answers by 35.Change the decimal answer to a percentage by moving the decimal two places to theright.

CALCULATING YOUR SCORE

Applied Technology • 123

How well did you do on the Posttest? If you scored 91% or higher, you have a reasonable chance to pass Level 5 of the Applied Technology assessment. Remember to use the IDEAL model when solving problems.

I = Identify the problemD = Define and represent the problem

boundariesE = Explore alternative approachesA = Act on a planL = Look at the result

Now, don’t be discouraged if you scored below 91%.Practice the exercises in this course – you can do it.Your enhanced work skills will pay off in the long run.Don’t forget to practice your problem-solving skills.

SUMMARY

Congratulations!

124 • Applied Technology

REFERENCE

EDWIN’S TEST–TAKING TIPS

Preparing for the test . . .

Complete appropriate levels of the WIN Instruction Solution self-study courses. Practice the exercises until you begin to feel comfortable solving problems.

Get a good night’s rest the night before the test and eat a good breakfast on test day. Your body (specifically your mind) works better when you take good care of it.

You should take the following items with you when you take the Applied Technology assessment: (1) pencils; pens are not allowed to be used on the test; it is a good idea to have more than one pencil since the test is timed and you do not want to waste time sharpening a broken pencil lead; and (2) your calculator; be sure your batteries are strong if you do not have a solar-powered calculator and that your calculator is working properly. Pencils will be provided for those who need them.

Allow adequate time to arrive at the test site. Being in a rush or arriving late will likely upset your concentration when you actually take the test.

About the test . . .

The test is comprised of approximately 32 multiple-choice questions. The questionscover four areas: thermodynamics, fluid dynamics, electricity, and mechanics. Somequestions will be presented as single questions while others may be in groups of twoand refer to specific figures or scenarios. You will not be penalized for wrong answers,so it is better to guess than leave blanks. You will have 45 minutes to complete thetest.

You will not be allowed to use scratch paper, but there is room in your assessmentbooklet to make calculations in solving.

Applied Technology • 125

REFERENCE

During the test . . .

Listen to instructions carefully and read the test booklet directions. Do not hesitateto ask the administrator questions if you do not understand what to do.

Pace yourself since this is a timed test. The administrator will let you know when youhave 5 minutes left and again when you have 1 minute remaining. Work as quickly aspossible, but be especially careful as you enter numbers into your calculator.

If a problem seems too difficult when you read it, skip over it (temporarily) and moveon to an easier problem. Be sure to put your answers in the right place. Sometimesskipping problems can cause you to get on the wrong line, so be careful. You mightwant to make a mark in the margin of the test, so that you will remember to go backto any skipped problems.

Since this is a multiple-choice test, you have an advantage answering problems thatare giving you trouble. Try to eliminate any unreasonable answers and make aneducated guess from the answers you have left.

If the administrator indicates you have one minute remaining and you have someunanswered questions, be sure to fill in an answer for every problem. Your guess isbetter than no answer at all!

If you answer all of the test questions before time is called, use the extra time to checkyour answers. It is easy to hit the wrong key on a calculator or place an answer on thewrong line when you are nervous. Look to see that you have not accidentally omittedany answers.

126 • Applied Technology

Dealing with test anxiety . . .

Being prepared is one of the best ways to reduce test anxiety. Remember to use thefive steps that we used in solving problems: identify the problem, define the problem,explore alternatives, act on a plan, and look at the result. Identifying several ways tosolve problems and following a systematic process should increase your confidenceand reduce anxiety.

Do not think negatively about the test. The story about the “little engine that could”is true. You must, “think you can, think you can, think you can.” If you prepareyourself by studying problem-solving strategies, there is no reason why you cannot besuccessful.

Do not expect yourself to know how to solve every problem. Do not expect to knowimmediately how to work the problems when you read them. Everyone has to readand reread problems when they are solving problems. So, don’t get discouraged; bepersistent.

Prior to the test, close your eyes, take several deep breaths, and think of a relaxingplace or a favorite activity. Visualize this setting for a minute or two before the test isadministered.

During the test if you find yourself tense and unable to think, try the followingrelaxation technique:

1. Put your feet on the floor.2. Grab under your chair with your hands. (Hope there are no surprises!)3. Push down with your feet and up on your chair at the same time - hold for 5

seconds.4. Relax 5 seconds (especially try to relax your neck and shoulders).5. Repeat a couple of times as needed, but do not spend the entire 45 minutes of

test trying to relax!

Studying with a partner is another way to overcome test anxiety. Encouragementfrom each other helps to increase your confidence.

REFERENCE

Applied Technology • 127

BASIC SCIENTIFIC PRINCIPLES

Applied Technology focuses on:• Principles related to power sources - for mechanical, electrical, thermal, and fluid

systems• Principles related to flow - for mechanical, electrical, thermal, and fluid systems• Principles related to pressure - for mechanical, electrical, thermal, and fluid systems• Principles related to resistance - for mechanical, electrical, thermal, and fluid systems

The basic scientific principles involved with energy sources, flow, pressure, and resistanceappear below:

Bernoulli’s principle: The faster the flow of air or fluid, the lower the pressure.

Boyle’s law: For a certain amount of gas, at a constant temperature, as the pressure (P)increases, the volume (V) of the gas decreases so that P times V is constant (k). (PV=k).

Charles’ law: For a certain amount of gas, at a constant pressure, as the absolutetemperature of the gas increases, the volume of the gas also increases. Mathematicallythis is: Volume (V) divided by temperature (T) equals a constant (k). V/T=k Thetemperature must be on an absolute scale that is in reference to absolute zero.

Hooke’s law: The greater the force exerted on an object, the more it will be moved. Forexample, the heavier the weight hanging from a spring, the more the spring will bestretched.

Newton’s laws of motion• An object will remain at rest or in uniform motion unless acted upon by an outside

force.• When a force acts upon an object, it changes the momentum of that object, and

this change is proportional to the applied force and to the time that it acts uponthe object.

• Every action (force) is followed by an equal and opposite reaction (force).

REFERENCE

128 • Applied Technology

Laws of Thermodynamics• Energy cannot be created or destroyed.• Heat energy always flows spontaneously from hot to cold.

Ohm’s law: Current is directly proportional to the voltage and inversely proportional tothe resistance.

Pascal’s law: Pressure added to a confined fluid at any point instantly appears equally atall other points and is always at right angles to the containing surfaces.

Generalizations that can be made about mechanics:

A machine is something that does work.

Work is done when a force causes an object to move.

Simple machines (gears, pulleys, inclined planes, levers, wheel and axle), which aredescribed below, make up compound (or complex) machines.

Compound machines include a bicycle, a rod and reel, a typewriter, a can opener, scissors,a hand drill, a car, a weight machine, and a treadmill.

Gears• The force that is applied to a driver gear is transferred to a driven gear.• When two gears of different sizes are meshed together, the smaller gear turns

faster (more rotations per minute) than the larger gear.• Gears that are meshed together move in opposite directions.• The direction and speed of the driver gear determines the speed and direction of

gears that are meshed with it.• Common applications of gears include bicycle sprocket chains, speedometers,

clocks, electric mixers, lawn sprinklers, and egg beaters.

REFERENCE

Applied Technology • 129

Pulleys• A pulley is a wheel with a rope, belt, or chain around it.• Pulleys change the direction of movement and make work easier.• Fixed pulleys change the direction that something is moved; they do not make

work easier.• Movable pulleys change the direction that something is moved and make work

easier.• The more pulleys in the system, the easier it is to do work (pull or lift an object).• The more pulleys involved in a system, the greater distance must be pulled, but

the easier it is to do work.• The thinner the windlass (winch), the easier it is to turn.• In two different sets of pulleys, if the wheels are connected by a shaft and the two

wheels on one pulley are the same size as the two wheels on the other pulley, theywill both turn at the same speed.

• Common pulley applications include crankshafts, sailboats, boat lifts, windowblinds, cranes, elevators, and escalators.

Inclined Planes• An inclined plane is a slanted surface that is used to raise or lower heavy objects

from one position to another.• Inclined planes help reduce the amount of force needed to do a given amount of

work, but require greater distance.• The steeper the plane, the more difficult the work.• Wedges are two back-to-back inclined planes.• Common applications of inclined planes include a screw, a bolt, a drill bit, a

clamp, a car jack, and a screw-on bottle top.

REFERENCE

130 • Applied Technology

Levers• A lever is a bar or rod that is free to move or turn on a fulcrum.• A lever multiplies force, but some distance must be given up.• The shorter the effort arm, the less force is attained and the greater distance is

attained.• The longer the effort arm, the more force is attained and the less distance is attained.• Examples of levers include scissors, a broom, a claw hammer, a nutcracker, a mop,

tongs, a crowbar, a can opener, tweezers, a baseball bat, boat oars, and a car jackhandle.

Wheel and Axle• A wheel and axle is like a spinning lever (an ice cream machine crank).• The center of the axle is the fulcrum.• The wheel is larger than the axle; for one rotation, a point on the edge of the

wheel travels a greater distance than a point on the axle. While the work doneby the axle and the wheel are the same, the greater distance traveled of thepoint on the edge of the wheel yields a smaller force at the edge of the wheelversus the edge of the axle.

• Common wheel and axle applications include a screwdriver, roller skates,a water-faucet handle, a bicycle pedal, a can opener, and a car steering wheel.

REFERENCE

Applied Technology • 131

Overview of Electricity

Electricity is the continuous flow of electrons, or current, from one atom to another. Noelectron flow will occur unless there is a pathway over which the electrons can move.This flow is similar to a water system, where pipes or hoses move water from storagetanks to where it is needed. In electrical wiring, the pathway through which electricalcurrent flows is called a circuit. A simple circuit consists of a power source, conductors,load, and a device for controlling current. Each is described below.

In buildings, the power source could be the electrical generating stations that pumpelectricity into residential and commercial buildings. However, other common sourcesof electrical power include small generators and batteries.

Conductors, or wiring, provide a path for the current so that it can travel from one pointto another.

A load is a device through which electricity produces work. For example, a lamp is a loadthat, when plugged in and turned on, produces light. Other examples of loads includeheaters, electric motors, and televisions.

Switches (on-off switches) control when electrical current flows through circuits. Fusesand circuit breakers are protective devices that control current by preventing too muchcurrent from flowing in the circuit, which would damage equipment. When an excessiveamount of electricity passes through them, fuses and circuit breakers “blow” to stop theflow of electricity through the circuit.

In a circuit, resistance lowers the amount of electrical energy available to do work. Bothwires and load affect resistance. It might be helpful to think of a similar situation with ahose that is connected to two sprinklers. As water passes through a hose, turns or kinksin the pathway cause friction (which is resistance) that results in a slower flow. In addition,when some of the water is diverted to the first sprinkler (which is a load), less water isavailable for use in the second sprinkler.

REFERENCE

132 • Applied Technology

There are two ways or methods of having current flow. Direct current flows in onedirection. In most cases, direct current is provided to equipment by batteries (flashlightsand portable radios). Alternating current flows in one direction, then reverses to theother direction. Alternating current is provided to equipment through electrical substationsin buildings. In the United States, common household current reverses itself 60 timesper second. This results in 120v 60 cycle AC. The international reference for cycles isdefined in hertz (one hertz = 1 cycle per second).

Measurement of Electric Current

The rate at which electricity flows is called amperage. It is measured in amperes. A 100-watt bulb requires a current of approximately 1 ampere to make it light up completely.Current flow is measured with an ammeter. Most electrically powered equipment indicatethe amount of current needed to operate it properly.

Measurement of Electrical Pressure

Pressure is applied to electrons to force them to move through a conductor and around acircuit. This pressure is measured in volts. The pressure, or voltage, is available in wiringcircuits all of the time - whether or not electrical equipment is being used. Voltage ismeasured with a voltmeter.

Calculation of Power

The amount of power derived from an electrical device or system is its wattage. In otherwords, it is the product obtained from electrical energy; it is the power that we put intouse. For example, the electric company sells electrical energy. Electrical energy or poweris measured in watts and can be calculated as follows:

For direct-current circuits: volts ××××× amperes = watts

For alternating-current circuits: volts ××××× amperes ××××× power factor = watts

NOTE: Power factors range from 0-1. Large equipment (an electric heater) may have apower factor as high as 1; small equipment (a small motor) may have a power factor aslow as .25.

REFERENCE

Applied Technology • 133

Ohm’s law

Ohm’s law is a simple formula used to describe the relationship between current (flow),voltage (pressure), and resistance of an electrical circuit. Each component interacts toaffect the operation of a circuit. In other words, because voltage pushes current througha resistance, a change in any of the components will result in a change in the others. Thefollowing three equations are Ohm’s law rearranged to solve for each of the quantities:

Current = Voltage ÷ Resistance I = E/Ramps = volts ÷ ohms An increase in voltage causes an increase in

electrical current flow. An increase in circuitresistance causes a decrease in electrical currentflow.

Voltage = Current ××××× Resistance E = I ××××× Rvolts = amps × ohms An increase in current causes an increase in

voltage. An increase in resistance causes anincrease in voltage.

Resistance = Voltage ÷ Current R = E/Iohms = volts ÷ amps

REFERENCE

134 • Applied Technology

Generalizations that can be made about electricity:

• The longer the wire, the greater the resistance; the thinner the wire, the greaterthe resistance.

• An increase in temperature of a wire causes an increase in resistance.

• An ordinary electrical cord has two wires; one for the flow of current from thepower source and the other for the return or ground.

• The voltage (pressure) and current (flow of electricity) directly affect how muchpower is available to do work. Less energy source or lower flow will result in lesselectrical power being produced.

• A series circuit has only one path for the flow of current. In a series circuit, objectsare placed one after another and the current flows through each of them insuccession. The current is the same throughout, however, and the voltage is dividedamong the objects in the circuit.

• In a parallel circuit, there are 2 or more paths, or branches, for the flow of current.The current will divide and flow through each of the paths simultaneously. Everybranch has the same voltage and - if the appliances are all the same - will have thesame amount of current. The total circuit resistance is less than any one branch.

• When batteries are connected in a series, the current is the same; the total voltageis the sum of the voltage of each battery. The terminals are connected +, -, +, -,and so on.

• When batteries are connected in parallel, the total current is the sum of the currentsin each battery; the total voltage is the same as that of one cell. The terminals areconnected +, +, +, and -, -, -.

REFERENCE

Applied Technology • 135

Generalizations that can be made about heat:

• Heat travels through conductors (metal) better than through insulators (wood).

• Dark-colored surfaces absorb more heat than light-colored surfaces.

• Rough or dull surfaces absorb more heat than smooth or shiny surfaces.

• When friction causes heat, the object that is in constant contact gets hotter thanthe movable object. (For example, the wood being cut gets hotter than the sawblade; car brake shoes get hotter than the wheel).

Generalizations that can be made about fluids:

Pressure• The amount of pressure exerted by a fluid depends upon the height and the density

of that fluid and is independent of the shape of the container that is holding thefluid.

• The deeper the fluid, the greater the pressure it exerts.

• The denser the fluid, the greater the pressure it exerts (salt water is denser thanfresh water).

• Fluids seek equilibrium - they seek their own level; a fluid will flow from a place ofhigh pressure to a place of low pressure.

• A fluid can never rise higher than its source without an external force (a pump).

REFERENCE

136 • Applied Technology

Evaporation• The higher a liquid’s temperature, the faster it will evaporate.

• The lower a liquid’s pressure, the faster the liquid will evaporate.

• The more area of liquid that is exposed to air, the faster the liquid will evaporate.

• The more circulation of air above a liquid, the faster the liquid will evaporate.

Boiling Point• Increased pressure on a liquid raises the liquid’s boiling point.

• Decreased pressure on a liquid lowers the liquid’s boiling point.

REFERENCE

Applied Technology • 137

BIBLIOGRAPHY

Adkinson, S., & Fleer, M. (Eds.). (1995). Science with reason. London: Hodder andStoughton Educational.

American Association for the Advancement of Science (1993). Benchmarks for scienceliteracy: A project 2061 report. New York: Oxford University Press.

American Association for the Advancement of Science (1990). Science for all Americans:A project 2061 report on literacy goals in science, mathematics, and technology. New York:Oxford University Press.

Bransford, J., & Stein, B. (1984). The IDEAL problem solver: A guide for improvingthinking, learning, and creativity. New York: W. H. Freeman and Co.

Ohio Department of Education (1994). Ohio’s competency-based science model: Scientificliteracy for the 21st century. Columbus, OH: State Board of Education.

National Council of Teachers of Mathematics (1993). Curriculum and evaluationstandards for school mathematics. Reston, VA: NCTM.

National Research Council (1996). National science educational standards. Washington,DC: National Academy Press.

The Secretary’s Commission on Achieving Necessary Skills (1992). Learning a living:A blueprint for high performance: A SCANS report for America 2000. Washington, DC:U.S. Department of Labor.

REFERENCE

138 • Applied Technology

ANSWERS TO POP QUIZ QUESTIONS

Page 17 — IDEAL Strategy• Identify the problem• Define and represent the problem• Explore alternatives• Act on a plan• Look at the result

Page 36 — As fluid depth increases, the pressure increases accordingly.

Page 64 — A circuit that has more than one path is called a parallelcircuit.

Worldwide Interactive Network, Inc.1000 Waterford Place Kingston, TN 37763

Toll-free 888.717.9461Fax 865.717.9461 www.w-win.com

WIN Career Readiness Courseware - ©2008 Worldwide Interactive Network, Inc. All rights reserved.